You are on page 1of 149

CHAPTER 1

DIVISION OF SCIENCE
Mathematics

 Arithmetic
 Plane and Analytic Geometry
 Statistics
 Algebra

Page 1 of 149
 Trigonometry
 Differential and Integral Calculus
 Differential Equation

Logic

 Inductive
 Deductive

Physical Science

 Physics
 Geology
 Astronomy
 Chemistry
 Meteorology

Life Science

 Anatomy
 Physiology
 Microbiology
 Pathology

Social Science

 Sociology
 Psychology
 Political Science
 Economics
 Anthropology

PHYSICS

Wave
 Acoustic “sound”
 Optics “light”

Electromagnetism
 Electricity
 Magnetism

Page 2 of 149
Mechanics
 Motion

Life Physics

Thermodynamics
 Heat
 Fluid

Atomic Physics

SIGNIFICANT FIGURES
 All non-zero digits are significant.
Ex. numbers 1, 2, 3, 4, 5, 6, 7, 8 & 9

 Zeros in between non zero digits are significant


Ex. 101 – 3 20002 – 5

 Zeros to the right of a non-zero digits in unexpressed decimal point are not
significant
Ex. 200 - 1

 Zeroes at the right of a non-zero digit in an express decimal point are significant.
Ex. 200.0 – 4

 Zeros at left of a non-zero digit but to the right of a decimal point are not
significant.
Ex. 0.01 - 1

Page 3 of 149
Rounding Off

 When the number to be rounded off is less than 5 change that number to zero “0”
to retain the preceding number.
Ex. 12.34 – 12

 When the number to be rounded off is greater than or equivalent to 5 change that
number to zero “0” and add one to the preceding number.
Ex. 98.76 – 99

SCIENTIFIC NOTATION
 It simplifies the writing, reading and computation of very large and small number
thereby decreasing the risk of errors.

Exponential
 Uses the power of ten
Writing as: N = ax10b
Where:
N = any given number
a = number having single non-zero digit to the left of a decimal point and
two decimal places.

RULES IN SIGNIFICANT NUMBER IN SCIENTIFIC NOTATION


a. Determine “a” – by shifting the decimal point of the original number to the left or
right, until one digit is to the left of it.
b. Determine “b” – by counting the number of decimal places the point has moved,
if it has been to the left “b” is positive if to the right “b” is negative.

Ex. 123456 – 1.23 x 105 left - +


0.0009876 9.88 x 10-4 right - -

BASIC CONCEPTS IN PHYSICS

Matter = Anything that occupied space and has mass.

Mass = Quantity of matter

Force = It is capable of changing the condition of rest or motion of a certain body.

Physical Quantity = Any number used to describe a physical phenomenon.


Ex. Height 5’2 – Quantity
2 Ways of Describing a Physical Quantity
a. Direct – which means measuring
b. Indirect – calculating, gather information

Page 4 of 149
Operational Definitions = Definitions that gives of procedure of measuring the quantity.

BASIC MATHEMATICAL OPERATIONS

1. Algebraic Methods
 Solving an unknown of a given operation
 Trigonometric functions and law
 Analytical method – interprets through graphs

Units and Problem Solving


Important Terms:

Unit = a quantity in terms of which another quantity is used.

Standard = a unit established to have a precise definition of a unit.

Standard Unit = if a unit becomes officially accepted

System of Units = a group of standard units and their combinations

International System of Units = the modernized version of the metric system which
includes the base quantities and derived quantities

SI Base Units = represented by standards

SI Derived Units = other quantities that may be expressed in terms of combinations of


the base units

Meter (m) = the SI unit for length, 1/10,000,000 of the distance from the North Pole to
the Equator along the meridian running through Paris

Kilogram (kg) = The SI unit of mass, specific volume of water but is now referenced to a
specific material standard The mass of a cylinder of a platinum, iridium, alloy, kept at the
international Bureau of Weights and Measurements in Paris, France

Second (s) = the SI unit of time, time required for cesium 133/133 CS to undergo
9192631770 vibration
MKS System = meter-kilogram-second

CGS System = centimeter-gram-second, Gaussian System

FPS System = foot-pounds-second, British Engineering System

Liter (L) = the nonstandard unit of volume

Page 5 of 149
Dimensional Analysis = a procedure by which the dimensional consistency of any
equation may be checked

Unit Analysis = using units instead of symbols in dimension analysis

Density (ρ) = ratio of mass to the volume

Conversion Factor = equivalent statements expressed in the form of ratios

Exact Number = those without any uncertainty or error

Measured Number = obtained from measurement processes and so generally have some
degree of uncertainty or error

Significant Figures (sf) = the number of reliably known digits it contains

Important Equations:

Density
ρ = m/V
Where:ρ = density
m = mass
V = volume

PREFIXES
21
Zetta x 10 Zepto x 10-21
Exa x 1018 Atto x 10-18
Peta x 1015 Femto x 10-15
Tera x 1012 Meter/Gram/Liter/Pascal Pico x 10-12
Giga x 109 x 100 Nano x 10-9
Mega x 106 Micro x 10-6
Kilo x 103 Milli x 10-3
Hecto x 102 Centi x 10-2
Deca x 101 Deci x 10-1

1m = 100 cm 1 kg = 2.2 lb
1m = 3.28 ft 1 hr = 60 min
1in = 2.54 cm 1 min = 60 sec
1ft = 12 in 1 mL = 1 cc
1 yd = 3 ft 1L = 1000 ml
1 mi = 5280 ft 1 kips = 1000 lbs
1 mi = 1.609 km
1 km = 0.62 mi

100 212

Page 6 of 149
o o
C F

1 32

9
° F= ° C +32
5

5
° C= ( ° F−32 )
9

100−0 212−32 100 180


= , = ,
C−0 F−32 C F−32

100 C 5 C
= , =
180 F−32 9 F−32
O
K = 273 + oC
O
R = 460 + oF

Example 1:

1. 40 km/hr  ft/sec =

40 km 1000 m 3.28 ft 1 hr
x x x =36 . 44 ft / s
hr 1km 1m 3600 s

2. 500g  lbs

1 kg 2.2 lbs
500 g x x =1 .1 lbs
1000 g 1 kg

3. 5 ft 5 in  m

1 ft
5∈ x = 0.42ft
12∈¿ ¿

1m
5.42 ftx =1 . 65 m
3.28 ft

4. 4000 mi → km

1 km
4000 mi x =6 , 451. 61 km
0.62 mi

Page 7 of 149
1.609 km
4000 mi x =6 , 436 km
1mi

5280 ft 1m 1 km
4000 mi x x x =6 , 439 . 02 km
1 mi 3.28 ft 1000 m

5. 1 metric ton to kilograms

1000 kg
1 metric ton x =1000 kg
1 metric ton

N = +Y

W = -x E = +x

S = -y
Note: The direction will just be represented as the angle from any two of the fixed axes
which the vector is enclosed.

Resolution of Vectors
- Graphical Method

- Rules in determining the magnitude and directions of “R”

1. Construct an accurate Cartesian plane.


2. Plot the first force using a given scale.
3. At the end of the first force construct another accurate C.P. and make sure that it
is parallel to the first C.P.
4. Plot the second force using the new Cartesian Plane and so on so forth.
5. Using an arrow connect the tail of the first force to the head of the last force and
label it “R” for resultant
6. To determine the magnitude of R simply measure the length of R using the given
scale.
7. To determine the direction of R, simply measure the angle with respect with the Y
axis.
Resultant Vectors

Page 8 of 149
- It is a vector whose effect is the same as all the component vectors put
together.

Case I: Two forces acting on an object in the same direction

Ex. Given F1 = 10N due E = 1cm


F2 = 20 N due E = 2cm

Scale 10 N = 1 cm

F1 = 10N/10 = 1cm
F2 = 20 N/10 = 2cm

R = 3cm due E
R = 30N, due E

Case II: Two forces acting on an object on opposite directions

F1 = 40N due E
F2 = 60N due W

Scale: 20N = 1cm


F1 = 40N/20N = 2cm
F2 = 60N/20N = 3cm
R = 1cm, due W
R = 20N, due W
Case III: Two force acting on an object perpendicular to each other.

Page 9 of 149
F1 = 30N E
F2 = 40N S

Scale: 10N = 1cm


F1 = 30N/10N = 3cm
F2 = 40N/10N = 4cm
R = 5cm, S37oE
R = 50N S 37 ° E

Case IV: Two or more forces acting on an object at random directions.

F1 = 40N due E
F2 = 40N due E
F3 = 40N NE

Scale: 20N = 1cm


F1 = 40N/20N = 2cm
F2 = 40N/20N = 2cm
F3 = 40N/20N = 2cm
R = 5.8cm, N76oE
R = 116N N 76 ° E

Ex.

Page 10 of 149
F1 = 50N 50o E of S
F2 = 50N 30o W of N

Scale: 20N = 1cm


F1 = 50N/20N = 2.5cm
F2 = 50N/20N = 2.5cm
R = 0.95cm, N48oE
R = 19N, N 48 ° E

ANALYTICAL METHODS

Case I: Just add and follow the direction of the forces.

F1 = 10 N due E
F2 = 30 N due E
R = 40 N due E

Case II: You just subtract and follow the directions of greater value.

F1 = 40N due E
F2 = 60N due W
R = 20N due W

Case III: Use Pythagorean Theorem and tangent.


R=√ F 2x + F 2y
−1 F x
θ=tan ( )Fy
Example:
F1 = 30N due E
F2 = 40N due S

R=√ 30 2+ 402 = 50N


−1 F x −1 30
θ=tan ( )
Fy
=tan ( )
40
= S 36.8 E
= 36.87 E of S
Case IV: Construct a Cartesian plane without a need of a scale plot the given vectors in
the Cartesian plane and use component method or sine and cosine law.

Page 11 of 149
Ex.1
F1 = 50N 50oE of S
F2 = 50N 30o W of N

R=√50 2+50 2−2(50)(50) cos 20


= 17.36 N

sin β sin 20
50
=
17.36
β=sin−1 ( 5017.36
sin 20
)=80.09

θ=180−( 80.09+50 )=N 49.91 ° E


Forces Direction Fx Fy
50N S50E +38.30 -32.14
50N N30W -25 +43.30
+13.30 +11.16

Solution:

F x : 50 sin 50=38.30=+38.30 F y: 50 cos 50=32.14=−32.14


50 sin 30=25=−25 50 cos 30=43.30=+43.30

R=√13.302 +11.16 2=17.36 N

13.3
θ=tan −1 ( 11.16 )=N 50 ° E

Inclined Axis

Page 12 of 149
F1 = 50#,1:2
F2 = 100#, 3:1 y : x = rise is to run
F3 = 155#, 3:2

100#

3 50#
1
2
2 3 1

155#

Forces # Fx Fy

50 50( √25 )=+ 44.7 50( −1


√5 )
=−22.36

1 3
100 (
√ 10 )
100 (
√ 10 )
100 =+ 31.67 =+ 94.87

−2 3
155 (
√ 13 )
155 (
√ 13 )
155 =−85.97 =+128.97

-9.63 +201.48
2 2
R=√ 9.63 + 201.48 =201.71 ¿
9.63
θ=tan −1 (
201.48 )
=N 2.74 ° W

3 DIMENSIONAL
Components:

F x =F m x , F y =F m y , F z=F m z , F=F m d

F 2=F2x + F 2y + F 2z

d 2=x 2 + y 2 + z 2 , d= √ x 2 + y 2+ z2

Page 13 of 149
Slope:
Fx F y Fz F
F m= = = =
x y z d
Direction:
Fx Fy Fz
cos θ x = , cos θ y = , cos θ z=
F F F

F x =FCos θx , F y =FCos θ y , F z=FCos θz ,

Given:
x=3
y=4
z=5
F = 100#

Req’d.: Components

Solution:
d 2=x 2 + y 2 + z 2 , d= √ x 2 + y 2+ z2
d= √3 2+ 42 +52 =√50=5 √ 2

100 20 20 √ 2
F m= = = =10 √ 2
5 √2 √ 2 2

F x =F m x =3 ( 10 √2 ) =30 √ 2=42.43 ¿

F y =F m y =4 ( 10 √ 2 )=40 √ 2=56.57 ¿

F z =F m z=5 ( 10 √ 2 )=50 √ 2=70.71¿

Fx F 42.43
cos θ x =
F F( )
; θ x =cos−1 x =cos−1
100 ( )
=64.89 °

Fy F 56.57
cos θ y =
F F( )
; θ y =cos−1 y =cos−1
100 (
=55.55 °)
Fz F 70.71
cos θ z =
F F( )
; θz =cos−1 z =cos−1
100 ( )
=45.00°

Coordinate = head - tail

x = right or left
y = upward or downward
z = forward or backward
Page 14 of 149
Example:
From (0, 12, 0)

P = 280#  (-4, 0, 6)
F = 210#  (6, 0, 4)
T = 260#  (-4, 0, -3)

Required:
a. Resultant
b. Components
c. Directions.
Y

Solution:
force x y z d= √ x 2 + y 2 + z 2 F F x =F m x F y =F m y F z =F m z
F m=
d
280 -4 -12 6 14 20 -80 -240 120
260 -4 -12 -3 13 20 -80 -240 -60
210 6 -12 4 14 15 90 -180 60
∑ ❑ -70 -660 120
θ 84.04 11.89 79.75

R=√70 2+660 2+120 2


R = 674.46 # left downward forward

Page 15 of 149
Unit Vectors = A unit vector is vector with magnitude of unit. Its purpose is only to
show direction in space.

SUM OF UNIT OF VECTORS

A = Axi + Ayj
B = Bxi + Byj

Let C = SUM OF A & B

C=A+B
= (Axi + Ayj) + (Bxi + Byj)
= (Ax + Bx)i + (Ay + By)j
C = Cxi + Cyj

Mag of C = Cx2 + Cy2

Difference of Two Vectors

Let D = A – B
= (Axi + Ayj) – (Bxi + Byj)
= (Ax - Bx)i + (Ay – By)j
D = Dxi + Dyj

Mag of D = Dx2 + Dy2

Products of Vectors
I. Scalar Product (Dot Product)

Page 16 of 149
The Scalar product of 2 perpendicular vectors is zero.

A ∙ B=AB cos
Cos 0 = 1

A.B = Ax Bx + Ay By + Az Bz

Note = ixi=1 ixj=0


jxj=1 ixk=0
kxk=1 kxj=0

II. Vector Product of 2 Vectors is zero when they are parallel.

A x B = AB sin 

AxB Ax Bx i Ax Bx
Ay By j Ay By
Az Bz k Az Bz
AxB= (Ay Bz – Az By)i + (Az Bx – Ax Bz – Ax Bz) jt
(Ax By – Ay Bx) k

Example:

A = -i + 2j – 5k
B = 2i + 3j – 2k

Mag of A

A = √ 12+ 22+5 2=√ 30

Mag of B

Page 17 of 149
B = √ 22+ 32+ 22=√ 17
C=A+B
C = i + 5j – 7k

Mag of C

C = √ 12+ 52+ 72=√ 75=5 √ 3

D=A-B
D =-3i – 1j – 3k

Mag of D

D = √ 32 +12+ 32=√ 19

Dot Product

A.B = (-1 x 2) + (2 x 3) + (-5 x -2)


= 14

Cross Product =

−1 2 i −1 2
2 3 j 2 3 = (-3k – 10j – 4i) – (-15i + 2j + 4k)
−5 −2 k −5 −2

AXB = 11i – 12j – 7k

Mag of AxB= √ 112 +122 +72= √ 314

A · B = ABcos θ
14
θ=cos−1 (
√ 30 x 17 )
=51.69

A x B = ABsinθ
314
θ=sin−1 √
(√ x 17
30 )
=51.69

Page 18 of 149
Name:
Course/Year/Section:

A. Count the number of significant figures and place the answer before the number and
transform the following to Scientific Notation
3 1. 0.000567 = 5.67 x 10−4
7 2. 6705001 = 6.71 x 106
5 3. 0.00090350 = 9.04 x 10−4
5 4. 84.650 = 8.47 x 10 1
7 5. 0.01425001 = 1.43 x 10−2

B. Perform the following operations. Final answer should be in scientific notation.

1. 0.03451 x 250 – 670.8 = −1.56 x 10−3


705 + 96.20  (0.35 x 0.00065)

2. (15430.0 4x 0.052) – 600 ¿ 1.74 x 10−2


0.00705 + 6208  (3550 x 0.00015)

3. (3165 x 5.35) – 0.0002643¿ 1.87 x 10−2


905000 + 0.0051  (0.46 x 0.0006708)

C. Convert the following.

1. 78.8 in to m

78.8∈¿

2. 1.53 m to ft

1.53 m ( 3.281 mft )=5.02 ft


3. 1.8 lbs to kg

1 kg
1.8 lbs ( 2.2lbs )=0.82 kg
4. 9500 mg to kg

9500 mg ( 10001 gmg )( 1000


1 kg
g)
=9.5 x 10 −3
kg

5. 4520 cm/s to ft/s

Page 19 of 149
cm 1 ft ft
4520 (
s 30.48 cm )
=148.29
s

6. 2.4g to cg

1 cg
2.4 g
( 10−2 g )
=240 cg

7. 45.2 oC to oF

45.2 ( 95 )+32=113.36 ℉
8. 77 oF to C

5
( 77−32 )=25 ℃
9

9. 20 mi/hr to m/s

mi 1.609 km
20
hr (
1mi )( 1000
1km
m
)( 1 hr
3600 s )=8.94
m
s

10. 1120 oF to oK

5
( 1120−32 ) +273.15=877.44 K
9

11. 97.75 oK to C

97.75−273=−175.25 ℃

12. -1 oC to K

−1+273=272 K

13. 87.9 oF to C

5
( 87.9−32 )=31.06 ℃
9

14. 101.97 ft/sec to m/hr

Page 20 of 149
ft 30.48 cm 1m
101.97
s( 1 ft )(
100 cm )( 3600 s
1 hr )
5
=1.12 x 10
m
hr

15. 5oF to R

460+ 5=465 R

D.1. F1 = 89N, 73o W of N


F2 = 77N, N 40o E

Scale: 40N= 1 cm
89 N
F1=
40 N
= 2.225 cm

77 N
F2=
40 N
= 1.925 cm

R= (2.4 cm) (40) = 96N

θ = N20˚W
Analytical

Forces Direction Fx Fy
89 N N 73 W -85.11 +26.02
77 N N 40 E +49.49 +58.99
-35.62 +85.01

R= √ (−35.62 )2+ ¿ ¿
R= 92.17 N

Page 21 of 149
θ=tan −1 ( 35.62
85.01 )

θ=N 22.73 ° W

2. F1 = 157# 46 E of S
F2 = 175# 56 E of N

Scale: 50#= 1 cm

157
F1= =3.14 cm
50

175
F2= =3.5 cm
50

R= (5.2) (50) = 260#

θ=S 88° E

Analytical

Forces Direction Fx Fy
157 S 46 E +112.94 -109.96
175 N 56 E +145.08 +97.86
+258.02 -11.2

R=√ (258.02)2+(11.2)2
R= 258.26#

θ=tan −1 ( 258.02
11.2 )

Page 22 of 149
θ=S 87.51° E

3. F1 = 155, N 68o E
F2 = 196N, N 72 W

Scale: 50N= 1 cm

155
F1= = 3.1 cm
50

196
F2= = 3.92 cm
50

R= (2.6) (50) = 130N

θ=N 19 ° W

Analytical

Forces Direction Fx Fy
155 N N 68° E +143.71 +58.06
196 N N 72° W -186.41 +60.57
-42.71 +118.63

R= √ (42.7)2+(48.63)2
R= 126 N

Page 23 of 149
42.7
θ=tan −1 ( 118.63 )
θ=N 19.8 ° W

4. F1 = 70N due W
F2 = 150N due N
F3 = 20N due E
F4 = 30N due S

Scale: 20N= 1 cm
70
F1= = 3.5 cm
20
150
F2= = 7.5 cm
20
20
F3= = 1 cm
20

R= (6.6) (20) = 132 N

θ=N 20 ° W

Analytical

Forces Direction Fx Fy
70N Due W -70
150N Due N +150
20N Due E +20
30N Due S -30
+50 +120

R= √ (−50)2 +(120)2
R= 130 N

Page 24 of 149
50
θ=tan −1 ( 120 )
θ=N 22.61 ° W

E.
From (3,6,-1)

A 100KN(-5,0,4)
To B 200KN(4,0,-5)
C 300KN(2,0,1)
Y

Force x y z d fm fx fy fz
100N -8 -6 5 5 √5 8.94 -71.52 -53.64 44.7
200N 1 -6 -4 √ 53 27.47 27.47 -164.82 -109.88
300N -1 -6 2 √ 41 46.88 -46.88 -281.82 93.76
∑❑ -90.93 -499.74 28.58
θ 79.70 10.80 86.78

R = √ −90.932 +−499.742 +28.582


= 508.75 KN left downward forward

Page 25 of 149
F.
A = -3i + 5j – 6k
B = 7i + 8j – 3k

Mag of A

A= √ 32 +52 +62 = √ 70 2 pts


Mag of B

B= √ 72 +82 +32 = √ 1222 pts


C = 4i + 13j – 9k2 pts

Mag of C

C = √ 4 2+ 132+ 92 = √ 266 2 pts

D = -10i – 3j - 3k2 pts

Mag of D

D = √ 102 +32 +32 = √ 118 2 pts

Dot Product

A.B = (-3x7) + (5X8) + (-6X-3) = 372 pts

Cross Product

AXB = 33i – 51j – 59k2 pts


Mag of AXB = √ 332 +512+ 592 = √ 71712 pts

37
 = cos
−1
(
√ 70 x √ 122 )
= 66.40° 2 pts

Page 26 of 149
θ=sin−1 ( √ 7084.68
x √122 )
= 66.40°

Chapter 2

Kinematics: Description of Motion


Important Terms:

Mechanics = the branch of physics concerned with the study of motion and what
produces and affects motion

Kinematics = deals with the description of motion of objects without consideration of


what causes the motion

Dynamics = analyzes the causes of motion

Motion = involves the changing of position

Distance = the total path length traversed in moving from one location to another

Scalar Quantity = one with magnitude only

Speed = the rate at which distance is traveled

Average Speed = the distance traveled divided by the total time elapsed in traveling that
distance

Instantaneous Speed = is how fast something is moving at a particular instant of time

Displacement = the straight line distance between two points along with the direction
from the starting point to the final position

Vector Quantity = has both magnitude and direction

Velocity = how fast something is moving and in what direction

Average Velocity = displacement divided by the total travel time

Page 27 of 149
Instantaneous Velocity = describes how fast something is moving and in what direction
at a particular instant of time.

Acceleration = time rate of change of velocity

Average Acceleration = the change in velocity divided by the time taken to make the
change

Instantaneous Acceleration = acceleration at a particular instant


Free Fall = objects in motion solely under the influence of gravity
Important Equations:

Average Speed
S = d/t
Where: S = speed
d = distance
t = time

Kinematic Equation for Linear Motion with Constant Acceleration


x = vt
Where:x = displacement
v = velocity
t = time

v = (vf + vi)/2
vf = vi + at
x = vit + ½at2
vf2 = vi2 + 2ax

Kinematic Equation Applied to Free Fall


y = vt
v = (vf + vi)/2
vf = vi + gt
y = vit + ½gt2
vf2 = vi2 + 2gy

Example 1: Sojourner could move at a maximum average speed of about 0.6m/min. At


this speed, how long would it take the rover to travel 3m to get to another rock to
analyze?

Given:
m 1min m
s=0.6 × =0.01
min 60 s s
d= 3m

Required:

Page 28 of 149
t= ?

Solution:
d
s=
t
m 3m
0.01 =
s t
3m
t= =300 s
m
0.01
s
Example 2: A jogger jogs from one end to the other of a straight 300m track(from point
A to point B) in 2.5min and then turns around and jogs 100 m back towards the starting
point (to point C) in another 1min. What are the jogger’s average speeds and average
velocities in going from A to B and from B to C after turning around?

Given: 300 m m
S AB= =2
dAB = 300m 150 s s
60 secs
t AB=2.5 minsx =150 s
1 min 100 m m
S BC = =1.67
dBC = 100m 60 s s
60 secs
t BC =1 minx =60 s
1min 300+100 400 m
S AC = = =1.9
150+60 210 s
Required:
SAB = ? 300 m m
V AB= =2
SBC = ? 150 s s
VAB = ?
VBC = ? −100 m m
V BC = =−1.67
60 s s
Solution:

300−100 200 m
V AC = = =0.95
150+60 210 s
Example 3: A couple in their sport utility vehicle is traveling 90km/h down a straight
highway. They see an accident in the distance, so the driver slows down to 40km/h in 5s.
What is the average acceleration of the SUV?

Given:
km 1hr 1000 m m
Vi=90 × × =25
hr 3600 s 1 km s

km 1 hr 1000 m m
Vf =40 × × =11.11
hr 3600 s 1 km s

t = 5secs

Page 29 of 149
Required:
a=?

Solution:

Vf −Vi 11.11−25 m
a= = =−2.78 2
t 5 s

Example 4: A drag racer starting from rest accelerates in a straight line at a constant rate
of 5.5m/s2 for 6s. What is the racer’s velocity at the end of this time? If a parachute
deployed at this time causes the racer to slow down uniformly at a rate of 2.4m/s 2, how
long will it take the racer to come to a stop?

Given:
m Solution:
Vi=0
s a . Vf =Vi+ at
m ¿ 5.5 ×6
a=5.5 2
s m
¿ 33 =Vi
t = 6secs s
m
a 2=−2.4
s2 b . Vf =Vi+ at
m 0=33+ (−2.4 ) t
Vf 2=0 m
s 33
s
t=
Required: m
2.4 2
a . Vf =? s
b . t =? t=13.75 s
Example 5: A motorboat starting from rest on a lake accelerates in a straight line at a
constant rate of 3m/s2 for 8s. How far does the boat travel during this time?

Given:
m Required:
Vi=0
s x=?
m
a=3 2
s Solution:
t=8 s x=v i t +0.5 a t 2
x=0+ 0.5× 3 ×82
x=96 m

Example 6: A body moves along a straight line following the equation x = 2t 2 – 5t meter
where t is in seconds. Solve the displacement of the body during the time interval from ti

Page 30 of 149
= 3 sec to t2 = 5s, what is the average velocity of the body, the instantaneous velocities at
this times, average acceleration and instantaneous acceleration?

Given: b. x 5=?
x=2 t 2 −5 t c. v́=?
t 3=3 s d. v3 =?
t 5=5 s e. v5 =?
f. a ave =?
Required: g. a inst=?
a. x 3=?

Solution:

a . x 3=2 ( 3 )2−5 ( 3 ) b . x 5=2 (5 )2−5 ( 5 )


¿ 18−15 ¿ 50−25
¿3m ¿ 25 m
c . ∆ d=25−3=22 m ∆d 22 m
v́= = =11
∆ t 5−3 s
dx
=v=4 t−5 instantaneous velocity
dt

d . v 3=4 ( 3 )−5 e . v 5=4 ( 5 )−5


v3 =12−5 v5 =20−5
m m
v3 =7 v5 =15
s s

f.
∆ v 15−7 m
a ave = = =4 2
∆ t 5−3 s
g..
dv m
=a=4 2 instantaneous acceleration
dt s

Example 7: The motion of a body along a straight line is described by the equation x =
0.2t3 + 3t2 + 2t – 5 meters, where t is in seconds. Solve the displacement of the body
during the time interval from t2 = 2sec to t4 = 4s, what is the average velocity of the body,
the instantaneous velocities at this times, average acceleration and instantaneous
acceleration?

Given: a. x 2=?
x=0.2 t 3+3 t 2 +2t−5 b. x 4 =?
t 2=2 s c. v́=?
t 4=4 s d. v 2=?
e. v 4=?
Required:

Page 31 of 149
f. a ave =? h. a 4=?
g. a 2=?

Solution:
a . x 2=0.2 ( 2 )3+3 ( 2 )2 +2 ( 2 )−5 b . x 4=0.2 ( 4 )3 +3 ( 4 )2+ 2 ( 4 ) −5
¿ 1.6+12+ 4−5 ¿ 12.8+ 48+ 8−5
¿ 12.6 m ¿ 63.8 m
c..
∆ d=63.8−12.6=51.2 m

∆ d 51.2 m
v́= = =25.6
∆ t 4−2 s
dx
=v=0.6 ( t )2 +6 t+ 2
dt

d . v 2=0.6 (2)2 +6 ( 2 ) +2
¿ 2.4+12+2 e . v 4=0.6(4 )2+ 6 ( 4 )+2
m ¿ 9.6+ 24+2
¿ 16.4
s
m 35.6−16.4 m
¿ 35.6 f . a ave = =9.6 2
s 4−2 s

dv
=a=1.2t +6
dt

g . a2=1.2 ( 2 )+ 6 h . a4 =1.2 ( 4 )+ 6
m m
¿ 8.4 2 ¿ 10.8 2
s s

Example 8: At the instant the traffic light turns green. An automobile that has been
waiting at the intersection starts ahead with constant acceleration of 2m/s 2. At the same
instant a truck, traveling with a constant speed of 18m/s overtakes and passes the
automobile. How far beyond its starting point does the automobile overtake the truck?
How fast is the automobile traveling when it overtakes the truck?

Given:
aC = 2m/s2 Required:
Vai = 0m/s X=?
aT = 0m/s2 Vfa = ?
VT = 18m/s

Solution:

Page 32 of 149
Xa = XT
1
x t=V i t + a t 2=V i t=18 t
2
1 1
x a=V i t+ at 2= a t 2=t 2
2 2

x t=x c
18 t=t 2
t=18 s

a . x c =t 2 =182=324 m

b . Vf =Vi+ at
m
¿ 2 ( 18 )=36
s
Example 9: What if the car is 20 ahead, the green light turns on.

XT

20m
m Xa

x t=x c +20
18 t=t 2+20
t 2−18 t+20=0
18± √ 182−4 ( 1 )( 20 ) 18 ±15.62
t= =
2 ( 1) 2
t 2=16.81 s
t 1=1.19 s

Example 10:
Time (s) Distance (m)
0 0

Page 33 of 149
1 2
2 3
3 5
4 8
5 12

Distance Versus Time Graph


14
12
10
8
Distance

6
4
2
0
0 1 2 3 4 5
Time

Velocity(m/s) Time (s) Acceleration (m/s2)

V1 = 2 1 A2 = -1
V2 = 1 2 A3 = 1
V3 = 2 3 A4 = 1
V4 = 3 4 A5 = 1
V5 = 4 5

Velocity Versus Time Graph


4.5
4
3.5
3
Velocity

2.5
2
1.5
1
0.5
0
0 1 2 3 4 5
Time

Page 34 of 149
Acceleration Versus Time Graph
2.5
2
Acceleration 1.5
1
0.5
0
0 1 2 3 4 5
-0.5
-1
-1.5
Time

Example 11:

Time, (sec) Distance, (m)


0 0
1 2
2 4
3 6
4 8
5 10

Page 35 of 149
Distance Versus Time graph
12

10

Distance 8

0
1 2 3 4 5 6
Time

Velocity, (m/s) Time, (s) Acceleration, (m/s2)


0
V1 = 2 1 A2 = 0
V2 = 2 2 A3 = 0
V3 = 2 3 A4 = 0
V4 = 2 4 A5 = 0
V5 = 2 5

Velocity Versus Time Graph


2.5

1.5
Velocity

0.5

0
0 1 2 3 4 5
Time

Page 36 of 149
Acceleration Versus Time Graph
2.5

Acceleration 1.5

0.5

0
0 1 2 3 4 5
Time

Example 12: A ball is dropped out of a window near the top of a building. If the ball
accelerates towards the ground at a rate of 9.81m/s2, what is its velocity when it has
fallen 4.0m?

Given: y=−4 m
m
Vi=0
s Required:
m Vf = ?
g=−9.81 2
s

Solution:
V f 2=V i2+ 2 gy
V f 2=0+2 (−9.81 ) (−4 )
V f =√ 78.48
m
V f =−8.86
s

Page 37 of 149
Example 13: A boy on a bridge throws a stone vertically downward with an initial
velocity of 14.7m/s towards a river below. If the stone hits the water 2s later, what is the
height of the bridge above the water?

Given:
m Required:
V i=−14.7
s h=?
t=2 s

Solution:
1
y=V i t+ g t 2
2
¿−14.7 2 )+ 0.5 (−9.81 ) (22)
(
y=−49.02 m
h=49.02 m

Example 14: If a lady grasps the falling ruler as quickly as possible, and the length of the
ruler below the top of the finger is noted. If on the average the ruler descends 18cm
before it is caught, what is the person’s average reaction time?

Given:
y=−0.18 m Solution:
m 1
V i=0 y=V i t+ g t 2
s 2
G = -9.81m/2 −0.18=0+0.5 (−9.81 ) t 2

Required:
t=?
t=
√−0.18
0.5 (−9.81 )
t=0.19 s
=√ 0.037

Example 15: A worker on a scaffold on a billboard throws a ball straight up. It has an
initial velocity of 11.2m/s when it leaves his hand at the top of the billboard. What is the
maximum height the ball reaches relative to the top of the billboard? How long does it
take to reach this height? What is the position of the ball at 2s?

Given: ymax
m
V i=11.2 t=?
s
V f =0 m/s

Required:

Page 38 of 149
a . y max =?
b . t =?
c . y @2 s=?

Solution:
a . V f 2=V i2+ 2 gy
2 2
0 =( 11.2 ) +2 (−9.81 )( y )
11.22
y= =6.39 m
2 ( 9.81 )

b . V f =V i +¿
0=11.2+ (−9.81 ) t
−11.2
t= =1.14 s
(−9.81 )

c . y =V i t +0.5 g t 2
y=11.2 ( 2 ) +0.5 (−9.81 )( 2 )2
¿+2.78 m

Name:
Course/Year/Sec:

Given:
X = 5t3 – 6t2 – 7t
t1 = 1sec and t3 = 3 secs

Required:
Solve for
a.) displacement,

Page 39 of 149
b.) average velocity
c.) instantaneous velocities
d.) average acceleration
e.) instantaneous acceleration

Solution:
a. X1= 5 (1)3-7 (1)
= -8 m 2pts

X3= 5 (3)3-6 (3)2-7 (3)


= 60m 2pts

∆´d 60−(−8) 68
b.v= = = =34 m/s 2 pts
d 3−1 2

c .V =15 t 2−12 t−7 2 pts

V 1=15(1)2−12 ( 1 )−7=−4 m/s 2 pts

V 3=15 (3)2−12 ( 3 )−7=92 m/ s 2 pts

∆ V 92−(−4) 96
d . a Ave= = = =48 m/ s2 2 pts
∆t 3 s−1 s 2

e .a=30 t−12 2 pts

a 1=30 (1 )−12=18 m/ s 2 2 pts

a 3=30 ( 3 )−12=78 m/ s2 2 pts

Chapter 3

Motion in Two Dimensions


Important Terms:

Unit Vector = has a magnitude of unity or one and indicates the vector direction

Page 40 of 149
Analytical Component Method = involves the resolving the vectors into rectangular
components and adding the components for each axis independently

Relative Velocity = relative to a reference frame

Projectile Motion = two dimensional, curvilinear motion is the motion of objects that are
thrown or projected by some means

Parabola = the path of motion of the projectile

Projectile = a body which is given an initial velocity and allowed to fall under the action
of gravity. A body set into space without any motive power it its own.

Trajectory = path of projectile.

Range = maximum horizontal distance travelled

Important Equations:

Components of Initial Velocity


V Xi=V iCosθ
V Yi =V i Sinθ

Components of Displacement
X =V Xi t + 1/2a x t 2
Y =V Yi t+1/2 aY t 2

Component of Velocity
V xf =V xi + ax t
V Yf =V Yi + aY t

Vector Representation
Magnitude Angle Form
C=√ C X 2+ CY 2
θ=tan −1 ¿

Component Form
C=C X X +C Y Y

R=V i t
−v i2 sin 2 θ
R=
g

Area of the Spandrel

Page 41 of 149
bh
A=
n+ 1
Where: b = base
h = height
n = degree of the curve

Example 1: If the diagonally moving ball has a velocity of 0.5m/s at an angle of 37Ο
relative to the x-axis, find how far it travels in 3.0s by using the x and y components

Given: Solution:
V = 0.5m/s V X =VCosθ=0.5 cos 37=0.4 m/s
θ = 37o V Y =VSinθ=0.5 sin 37=0.30 m/s
t = 3s d
V = , d=Vt
t
Required:
X=? X =V X t=0.4 ( 3 )=1.2 m
Y=? Y =V Y t=0.3 ( 3 )=0.9m
Example 2: Suppose that the ball has an initial velocity of 1.5m/s along the x-axis and
starting at 0s, receives an acceleration of 2.8m/ s2 in the y direction. What is the position
of the ball 3s after 0s? What is the velocity of the ball at that time?

Given: Required:
V ix =1.5 m/s a. X3 = ?
a y =2.8 m/ s 2 Y3 = ?
t = 3s b. V3 = ?

Solution
X 3 =V ix t+ 0.5 ax t 2=1.5 ( 3 )+ 0.5 ( 0 )( 3 )2 =4.5 m
Y 3=V iy t+0.5 a y t 2=0+ 0.5 ( 2.8 )(3)2 =12.6 m

V fx 3=V ox 3+ a x t=1.5+0 ( 3 ) =1.5 m/ s


V fy 3=V oy 3 +a y t=0+2.8 ( 3 )=8.4 m/s
V f 3=√ 1.52+ 8.42=8.53 m/s
Example 3: Consider yourself in car A traveling along a straight, level highway with a
speed of 75km/h. Another car B travels at a speed of 90km/h. Find the differences in the
velocities VBA = VB – VA the other car travels in the same direction in front of you and
approaching you travelling in the opposite direction.

Given: Required:
VA = 75km/h V BAsame =?
VB = 90km/h

V BAopposite =? Solution:
V BAs ame :V b −V a=90−75=15 km/hr

Page 42 of 149
V BAopposite : V b −V a=−90−75=−165 km/hr

Example 4: The current of a 500m wide straight river has a flow rate of 2.55km/h. A
motorboat that travels with a constant speed of 8.0km/h in still water crosses the river. If
the boat’s bow points directly across the river toward the opposite shore, what is the
velocity of the boat relative to the stationary observer sitting at the corner of the bridge?
How far downstream will the boat’s landing point be from the point directly opposite its
starting point? What is the distance traveled by the boat in crossing the river?

Given:
w = 500m Required:
2.55 km 1 hr 1000 m a . V R =?
V C= × × =0.71 m/ s
hr 3600 s 1km b.. x = ?
8 km 1 hr 1000 m c.. d = ?
V B= × × =2.22 m/s
hr 3600 s 1 km
Solution:
a.) V R=√ 2.222 +0.712=2.33 m/s

θ=tan −1 ( 0.71
2.22 )
=N 17.74 ° E
500m

X
b.) tan17.74=
500
X =500 tan 17.74=159.96 m

c.) d= √5002 +159.962 =524.96 m

Example 5: An airplane with an air speed of 200km/hr flies in a direction such that with
a west wind of 50km/hr blowing, it travels in a straight line northward. To maintain its
course due north, the plane must fly at an angle. What is the speed of the plane along its
northward path?

Given:
V A =200 km/h
V W =50 km/h

Required:

Page 43 of 149
a . V R =?
b . θ=?

Solution:
a . V R =√ 2002−502 =193.65 km/hr

50
b . θ=sin−1 [ ]=N 14.48 ° W
200

Example 6: The Coordinates of a body moving in the xy plane are given as function of
time by: x = 3t2 – 4t + 1 & y = 4t2 – 4, x & y are in the m & t in sec.
t2 = 2s & t3 = 3s.

Given:
x = 3t2 – 4t + 1
y = 4t2 – 4
t2 = 2s
t3 = 3s

Required:
a. Determine the position of the body during these times
b. What is the average velocity of the body during the time interval?
c. What is the instantaneous velocity of the body during the time interval?
d. What is the average acceleration of the body during the time interval?
e. What is the instantaneous acceleration of the body?
f. What are the velocities and accelerations of a body at the instant it possess the x and y
– axes.

Solution:
a . x=3 t 2 – 4 t +1 y=4 t 2−4
x 2=3(2)2−4 ( 2 ) +1=5 m y 2=4( 2)2−4=12m
x 3=3(3)2−4 ( 3 ) +1=16 m y 3=4( 3)2−4=32 m

b . ∆ x=16−5=11 m
∆ y =32−12=20 m ∆ d 22.83 m
v= = =22.83 m/s
∆ d=√ 112 +202=22.83 m ∆t 1s

dx dy
c. =V x =6 t−4 =V y =8 t
dt dt

Vx 2=6 ( 2 )−4=8 m/s Vy 2=8 (2 )=16 m/s


Vx 3=6 ( 3 )−4=14 m/s Vy 3=8 (3 )=24 m/ s

V 2= √82 +16 2=17.89 m/s V 3= √14 2+ 242=27.7 m/ s

Page 44 of 149
d . ∆ V x =14−8=6 m/ s
∆ V y =24−16=8 m/s ∆ V 10
ā= = =10 m/s 2
2 2
∆ V =√ 6 +8 =10 m/s ∆t 1

dV x 2 dV y 2
e . A x= =6 m/ s A y= =8 m/s
dt dt

∴ a= √ 6 2+ 82=10 m/s 2

f.

(0,y)

(x,0)

Y=0
4 t 2−4 X=0
0=
4 0=3 t 2−4 t +1=( 3 t−1 ) (t−1)
2
t −1=0=( t+ 1 ) (t−1) 1
t 1= s
t=−1 s : no negative 3
t=1 s t 2=1 s
V X 1 =6 t−4=6−4=2m/s
1 V Y 1=8 t=8 m/s
V 1 =6 t−4=6
X
3
3 ()
−4=−2 m/s 8
V 1 =8 t = m/s
Y
3
3
V 1= √22 +82=8.25 m/ s 2

Constant acceleration = 10m/s2



V 1= 22 +
8
[]
3
=3.33 m/s

Example 7: Suppose that the ball is projected from a height of 25m above the ground and
is thrown with an initial horizontal velocity of 8.25m/s. How long is the ball in flight
before striking the ground? How far from the building does the ball strike the ground?

Given:
V ix =8.25 m/s
Y = 25m

Required:
t=?

Page 45 of 149
x
x=?

Solution:
a . y=V iy t +0.5 ¿2
−25=0+0.5(−9.81)t 2
2(25)
t=
√ 9.81
=2.26 sec

b . X =R=V ix t=8.25 ( 2.26 )=18.65 m

Example 8: A golf ball is hit off by the tee with an initial velocity of 30m/s at an angle of
35o to the horizontal. What is the maximum height reached by the ball? What is its
range?

Given: Required:
V i=30 m/ s a . y max =?
θ=35° b . R=?

Solution:
V ix =V i cos θ=30 cos 35=24.57 m/s
V iy =V i sinθ=30 sin 35=17.21 m/s

a . V 2fy =V 2iy + 2 gy
ymax
0=17.212+ 2 (−9.81 ) y
−(17.21)2
y= =15.1 m
2(−9.81) x

b . V fy =V iy + ¿
0=17.21+ (−9.81 ) t
−17.21
t= =1.75 s
−9.81
t T = 2(1.75) = 3.5sec
R= V ix t = 24.57(3.5) = 86.00m
2
−Vi 2 sin 2 θ −(30) sin (2 x 35)
R= = =86.21m
g −9.81
Example 9: The college basketball game is close. The home team trails by two points,
and 1s remains in the game. The home town guard takes a jump shot. The ball is
launched at the angle of 60o relative to the horizontal, with a speed of 10m/s and an initial
height of 3.05m above the floor, the same height as the hoop. The ball goes in and the
crowd goes wild. Basket was made from outside a semi circle of radius 6.02m from
beneath the center of the hoop are given three points, whereas those made from inside the
semicircle are worth two points. Did the home team win or did the game go into
overtime in a tie? Suppose the home guard stops at a distance of 7m from a point directly
below the point nearest to him on the hoop and takes a set shot with the same launched

Page 46 of 149
speed as in the first but an angle of 30 o and from an initial height of 1.8m above the floor.
Is there a possibility of him making the shot?

Given: Win or lose?


1st scenario
t=1 s 2nd scenario
θ=60 ° θ=30°
V i=10 m/s V i=10 m/s
h=3.05 m h=1.8 m
r =6.02m r =6.02m
Required:
Required: Win or lose?

Solution:
a.)V iy =10 sin 60=8.66 m/ s
V fy =V iy +¿
0=8.66±9.81t
8.66
t= =0.88 s
9.81
t T =2 ( 0.88 ) =1.76 s
V ix =V i cos 60=5 m/s

R=V ix t=5 (1.76 )=8.8 m


3point area = win

b.. V ix =10 cos 30=8.66 m/s


V iy =10 sin 30=5 m/s

R=V ixt
7=8.66(t )
t=0.81 s

y=V iy t+0.5 g t 2
y=5 (0.81)+ 0.5(−9.81)( 0.81 )2 =0.83 m

hremaining =3.05−1.8=1.25 m≠ 0.83 m: no shot


Example 10: A projectile has a speed of 18m/s. In it is desired to reach a target of 31m
away at the same elevation what should be the angle of projection?

Given:
V i=18 m/s
R = 31m

Required:

ymax Page 47 of 149

X = 31m
θ=?

Solution:
−V 12 sin 2θ
R=
g

−182 sin 2θ
31=
−9.81

31 ( 9.81 )

θ=
sin−1 (
182 )
=34.9 °
2
Example 11: Solve for the distance travelled after 1s and 2s.

3m/s2

A1 A2

1s 2s

bh
Area of Spandrel=
n+1

1(3) 3
A1= = =1.5 m/s
1+1 2

1(3)
A2= =3 m/s
0+1

V 1 = A1 = 1.5
V 2= A1+ A2 = 1.5+3 = 4.5

4.5m/s

Page 48 of 149
1.5m/s

bh
A = n+1

1(1.5) 1.5
A1 =
2+ 1
= 3
= 0.5 m

1(1.5) 1.5
A2 =
0+ 1
= 1 =
1.5 m

1(3) 3
A3 =
1+ 1
= 2
= 1.5 m

S1 = A1 = 0.5 m

S2 = A1+ A2+ A3 = 0.5+1.5+1.5= 3.5 m

Chapter 4

Force and Motion

Page 49 of 149
Important Terms:

Force = something that is capable of changing an object’s state of motion

Net (Unbalanced) Force = is the vector sum or resultant of all the force acting on an
object or system.

Inertia = the natural tendency of an object to maintain a state of rest or to remain in


uniform motion in a straight line

Newton’s First Law of Motion (Law of Inertia) = in the absence of an unbalanced


applied force, a body at rest remains at rest and a body already in motion remains in
motion with a constant velocity

Newton (unit) = the SI unit of force

Newton’s Second Law of Motion (Cause and Effect) = The acceleration of an object is
directly proportional to the net force acting on it and inversely proportional to its mass.
The direction of the acceleration is in the direction of the applied net force.

Weight = the only net force acting on a body

Newton’s Third Law of Motion (Action-Reaction) = For every force (action), there is
an equal and opposite force (reaction)

Normal Force = the force a surface exerts on an object

Free-body Diagram = represents an object as a particle or point sized mass and shows
all the forces acting on that particle.

Translational Equilibrium = the vector sum of the forces, or the net force is zero, so the
object either remains at rest or moves with a constant velocity

Force of Friction = refers to the resistance to motion that occurs whenever two materials
are in contact with each other.

Static Friction = the frictional force is sufficient to prevent relative motion between
surfaces

Kinetic (Sliding) Friction = occurs when there is a relative motion at the interface of the
surfaces in contact.
Rolling Friction = Occurs when one surface rotates as it moves over another surface but
does not slip or slide at the point or area of contact.

Coefficient of Static Friction = the ratio between the static frictional force and the
normal force

Page 50 of 149
Coefficient of Kinetic Friction = the ratio between the kinetic frictional force and the
normal force

Air Resistance = resistance force acting on an object as it moves through air

Terminal Velocity = refers to the maximum constant velocity

Important Equations:

Newton’s Second Law


F=ma
Where: F = force
m = mass
a = acceleration

Weight
w=mg
Where w = weight
g = acceleration due to gravity, 9.81m/s2

Component Form of Newton’s Second Law


F X X + F Y Y =ma X X +maY Y

Condition for Translational Equilibrium


ΣF=0 , Σ F X =0 , Σ F Y =0

Force of Static Friction


f S =μ S N
Where: fS = static frictional force
μS = Coefficient of static friction
N = normal force

Force of Kinetic (Sliding) Friction


F K =μ K N
Where: fK = kinetic frictional force
μK = Coefficient of kinetic friction
N = normal force

Example 1: A tractor pulls a loaded wagon with a constant force of 440N. If the total
mass of the wagon and its contents is 275kg, what is the wagon’s acceleration?

Given:
F = 440N
a=?
Page 51 of 149

F = 440N
m = 275kg
m = 275kg

Required:
a=?

Solution:
∑ F=ma
∑ F 440 N
a= = =1.6 m/s
m 275 kg

Example 2: A student weighs 588N. What is her mass?

Given:
W = 588N Solution:
W =mg
Required: −588 N=m(−9.81m/ s2 )
m=?

588 kg m/s 2
m= =59.9 kg
9.81 m/ s2
Example 3: Two blocks with masses 2.5kg and 3.5kg, rest on a frictionless surface and
are connected by a light string. A horizontal force of 12N is applied to the 2.5 kg. What
is the magnitude of the acceleration of the masses? What is the magnitude of the tensile
force in the string?

Given: F=12 N
m 1=2.5 kg
m 2=3.5 kg Required:
m T =6 kg a . a=?
b . T =?
3.5kg 2.5kg
Solution:
a . ∑ F=ma
12 N =(6)a
a=2 m/s2

b . F=ma 3.5kg
T =3.5 ( 2 )=7 N

Example 4: A block of mass 0.50kg travels with a speed of 2.0m/s in the x-direction on a
flat, frictionless surface. On passing through the origin, it experiences a constant force of
3.0N at an angle of 60o relative to the x-axis for 1.5s. What is the velocity of the block at
the end of this time?

Page 52 of 149
Given: F
F=ma ; a=
m=0.5 kg m
V ix =2 m/s
F=3 N F x 1.5 2
aX= = =3 m/ s
θ=60 ° m 0.5
t=1.5 s
F y 2.60 2
a y= = =5.2 m/ s
Required: m 0.5
V f =?
V fx =V ix + a X t=2+3 ( 1.5 )=6.5m/ s
Solution: V fy =V iy +a y t=5.2 ( 1.5 )=7.78 m/s
F y =3 sin 60=2.60 N
F x =3 cos 60=1.5 N

V f =√ 6.52 +7.792=10.15 m/s


Example 5: A car traveling at 72km/h along a straight, level road is brought uniformly to
a stop in a distance of 40.0m. If the car weighs 8.80 x 103N, what is the breaking force?

Given:
72 km 1hr 1000 m
V i= × × =20 m/ s
hr 3600 s 1 km
V f =0 m/s
d=40 m
W =8.80 x 103 N

Required:
F=?

Solution:
v f 2=v i2+2 ax 3.5kg
a
02 =202+ 2 a ( 40 ) F=?

−202 −400
a= = =−5 m/s 2
2(40) 80

∑ F=ma
3 2
W −8.80 ×10 N ( −5 m/s )
∑ F= a= =−4485.22 N
g −9.81 m/s 2
Example 6: The atwood machine consists of two masses suspended from a fixed pulley
with 0.55kg and 0.80kg. What is the acceleration of the system? What is the magnitude
of the tension in the string? Consider the pulley to be frictionless and the masses of the
string and the pulley to be negligible.

Given: W 1=0.8 kg ×9.81 m/s 2=7.84 N

Page 53 of 149
W 2 =0.55 kg ×9.81 m/s 2=5.40 N a . a=?
b . T =?
Required:
Solution: a
a

∑ F=ma
T −W =ma T T

T −5.40=0.55 a

∑ F=ma 0.80kg

T −W =m(−a)
T −7.84=−0.80 a
0.55kg W1
W2
(T −5.40=0.55 a)
−( T −7.84=−0.80 a)
2.44=1.35 a
a=1.81 m/s2

T =0.55 ( 1.81 ) +5.40=6.40 N

Example 7: A force of 15N is applied at an angle of 30 o to the horizontal on a 0.75kg


block at rest on a frictionless surface. What is the magnitude of the resulting acceleration
of the block? What is the magnitude of the normal force?

Given: Required:
F=15 N a=?
θ=30° N F=?
m=0.75 kg

Solution: F = 15N
0.75kg θ =30o

a . ∑ F=ma F y =15 sin 30=7.5 N


15 cos 30=0.75 kg( a)
15 cos 30 ∑ F y =0 ↑+¿
a= =17.32m/ s2
0.75 N F +W + F y =0
N F−7.36+ 7.5=0
b . W =0.75 (−9.81 )=−7.36 N N F=−7.5+ 7.36=−0.14 N

Example 8: Two masses are connected by a light string running over a light pulley of
negligible friction. The 5kg is on a frictionless 20o inclined plane and the other, 1.15kg is
freely suspended. What is the acceleration of the masses?

Given:
W 1=5 kg ( 9.81 )=49.05 N

Page 54 of 149
W 1 x =49.05 sin 20=16.78
θ=20°
W 2 =1.15 ( 9.81 )=11.28 N
a
a
Required:
a=? T
5kg
Solution:
∑ F=ma 1.15kg
−T + W x =5 a eqn .1 T

T −W 2 =1.15 a eqn .2
W2

Wx
(T −11.28=1.15 a)
+(−T + 16.78=5 a)
5.5=6.15 a
a=0.89 m/s 2

Example 9: A 3kg sign hangs in a wall in the room. What is the minimum tensile
strength necessary for the cord that is used to hang the sign?

Given:
m=3 kg
T T
Required:
T =?

Solution:
∑ F y =0 3kg

T + W =0
T =−W =mg=−3 (−9.81 )=29.43 N
W
T =29.43 N

Example 10:
T1T2

30o 30o

30o 30o

Page 55 of 149
W
m = 100kg
∑ F x =0+ →
−T 1 x + T 2 x =0
−T 1 cos 30+ T 2 cos 30=0
T 1=T 2 eqn 1

∑ F y =0+↑
T 1 y +T 2 y −W =0
T 1 sin 30+T 2 sin 30=mg eqn 2
9.81m
2 T 1 sin 30=100 kg
( s2 )
T 1=T 2 =981 N

Example 10: If the coefficient of static friction between the 40kg crate and the floor is
0.65, with what horizontal force must the worker pull to move the crate? If the worker
maintains that force once the crate starts to move and the coefficient of kinetic friction
between the surfaces is 0.5, what is the magnitude of the acceleration of the crate?

Given:
W =40( 9.81)=392 N ¿ N f
μs =0.65
μk =0.5
40kg
a
Required: P=?
a . F=? fs
b . a=?

Solution:
F s=μ s N F
F s=0.65 ( 392.4 )=225.06 N=P a=?

F k =μ K N F =0.5 ( 392.4 )=196.2 N


40kg 40kg
P
∑ F=ma
fk
255.06−196.2=40 a fs

a=1.47 m/s 2

Example 11: A worker pulling a crate applies a force at an angle of 30 o to the horizontal.
How large a force must he apply to move the crate?

Page 56 of 149
Given:
W =40 ( 9.81 )=392.4 N Required:
θ=30° F=?

Solution:
∑ F y =0+↑
N f + W + P y =0 W
a
N f −392.4+ Psin 30=0 P=?
N f =392.4−Psin 30
θ = 30
F s=μ N F=0.65 ( 392.4−0.5 P ) 40kg fs

∑ F=ma
∑ F h=0+→ NF
P X −F=0

Pcos 30−0.65 ( 392.4−Psin 30 ) =0


Pcos 30−255.06 +0.33 P=0
255.06
P= =213.26 N
cos 30+ 0.33

Example 12: A block slides with a constant velocity down a plane inclined at 37o to the
horizontal. What is the coefficient of kinetic friction between the block and the plane?

Given: Required:
θ=37 ° μk =?

Solution:
W =mg Wx a
W x =mgsin 37 W
W Y =mgcos37=N f Wy

F=μ N f =μmgcos37
∑ F X =0+→
W x −F=0
mgsin 37−μmgcos37=0 fs
μmgcos 37=sin 37 (mg) NF
37o
mgsin 37
μ=
mgcos 37
μ=tan 37=0.75

Name:
Course/Year/Sec:

1ST LAW : Solve for the tensions in the cables.

Page 57 of 149
1.

∑ F y =0+↑
T −W =0
W = 100N

T =W =100 N

2.
∑ F y =0+↑
T 1 y −W =0
30o
T 1 sin 30=W
T 1 sin 30=100
100
T 1= =200 N
sin30
100N

∑ F x =0+ →
T 2−T 1 X =0
T 2=T 1 cos 30
T 2=200(cos 30)=173.21 N

3.
∑ F h=0+⟼
T 2 X −T 1 X =0
T 2 cos 40=T 1 cos 30 30o 40o
T 1 cos 30
T 2= eqn 1
cos 40

∑ F v =0+↑
T 2 y +T 1 y −W =0 m = 200kg
T 2 sin 40+ T 1 sin 30−200 ( 9.81 )=0
T 1 cos 30
( sin 40 ) +T 1 sin 30=1962
cos 40
T 1( tan 40) ( cos 30 ) +T 1 sin 30=1962
T 1(( tan 40) ( cos 30 )+ sin 30)=1962
1962
T 1=
((tan 40) ( cos 30 ) +sin 30)
T 1=1599.44 N
T 2=1808.19 N

2ND LAW EXAMPLES


Solve for the acceleration.

Page 58 of 149
1.
ΣF = ma
P=ma 10kg P = 100N
100=10 a
100
a= =10 m/ s2
10

2. T = 150N
ΣF = ma 15o
P+T x =ma
100+150 cos 15=ma 10kg P = 100N
150 N cos 15+100=10 (a)
244.89
a= =24.49 m/ s2
10

3.
ΣF = ma
P−T x =ma
T = 150N
100−150cos 15=10 a
−44.89 15o
a= =−4.49 m/s2
10 10kg P = 100N

4.
∑ F x =ma
10k
P+W x =ma g
100+WSin 10=ma
P = 100N
10 ( a ) =100+98.1 sin10
117.03
a= =11.70 m/s 2
10

5.
∑ F x =ma 10k
10o
g
- P+W x =ma
−100+WSin 10=ma P = 100N
10o
10 ( a ) =−100+98.1 sin 10
−82.97
a= =−8.30 m/ s 2
10

6.
∑ F x =0+ x
P+W x + T x =ma

Page 59 of 149
−100 N +10 ( 9.81 ) (sin 10)+50 cos 30=10 a
−39.66
a= =−3.97 m/s 2
10 T = 50N

30o

10k
g
10o P = 100N

7.

∑ F x =0+ x
P+W x + T x =ma
100 N +10 ( 9.81 ) (sin 10)+50 cos 30=10 a
160.34
a= =16.03 m/s 2
10 T = 50N

30o

10k
g
10o P = 100N

Friction: Solve for Normal Force, Frictional Force and Acceleration.


1.
μ = 0.15
Solution:
∑ F v =0+↑
N F−W =0 10kg P = 100N
N F=W =10 ( 9.81 )=98.1 N

F=μ N F =0.15( 98.1)=14.72 N

∑ F=ma
P−F=ma
100−14.72=10 a

Page 60 of 149
85.28
a= =8.53 m/s 2
10

2. μ = 0.1

L X =200 cos 15=193.19 N L= 200N


LY =200 sin 15=51.76 N 15o

W = 100N
∑ F v =0+↑ P = 50N
N F−W −LY =0
N F=W + LY
N F=100+ 51.76=151.76 N

F=μ N F =0.1(151.76 N )=15.18 N

∑ F h=+→
F h=P−L X
F h=50−193.19=−143.19 N

∑ F=ma
F h+ F=ma
100
−143.19+15.18= a
9.81
(−128.01 )( 9.81 )
a= =−12.56 m/s 2
100

3. μ = 0.09

W =10 ( 9.81 )=98.10 N


W x =WSinθ=98.10sin 15=25.39 N
W y =WCosθ=98.10 cos 15=94.76 N

∑ F y =0+ y
N F−W y =0
10k
N F=W y =94.76 N g
15o P = 100N

F=μ N F =0.09 ( 94.76 )=8.53 N

∑ F x =+ x
F x =−P+W X
F h=−100+25.39=−74.61 N

∑ F=ma
F h+ F=ma

Page 61 of 149
−74.61+8.53=10 a
−66.08
a= =−6.61 m/s 2
10

4. μ = 0.2 W y =WCosθ=98.10 cos 20=92.18 N


W =10 ( 9.81 )=98.10 N T x =TCosθ=150 cos 20=140.95 N
W x =WSinθ=98.10sin 20=33.55 N T y =TSinθ=150 sin 20=51.30 N

∑ F y =0+ y
N F−W y −T y =0 T = 150N
N F=W y +T y =92.18+51.30
N F=143.48 N 20o

F=μ N F =0.2 ( 143.48 )=2 8.70 N 10k


g
20o P = 100N
∑ F x =+ x
F x =−P+W X +T X
F h=−100+33.55+140.95=74.5 N

∑ F=ma
F h−F=ma
74.5−28.70=10 a
45.80
a= =4.58 m/ s2
10

5. μ = 0.15 W y =WCosθ=294.30cos 15
W =30 ( 9.81 )=294.30 N W y =284.27 N
W x =WSinθ=294.30 sin 15 T x =TCosθ =350 cos 10=344.68 N
W x =76.17 N
T y =TSinθ=350 sin 10=60.78 N
∑ F y =0+ y
N F−W y −T y =0
N F=W y +T y =284.27+60.78 T = 350N
N F=345.05 N
10o
F=μ N F =0.15 ( 345.05 )=51.76 N
30k
g
∑ F x =+ x 15o P = 500N
F x =−P+W X +T X
F h=−500+76.17+344.68=−79.15 N

∑ F=ma
F h−F=ma

Page 62 of 149
−79.15+51.76=30 a
−27.39
a= =−0.91 m/s 2
30

Chapter 5
Page 63 of 149
Work and Energy
Important Terms:

Work = the product of the magnitudes of the displacement and the component of the
force parallel to the displacement

Joule (unit) = the SI unit of Newton meters

Spring Constant (Force Constant) = constant of proportionality

Kinetic Energy = the energy of motion

Work-Energy Theorem = the net work done on a body by an external net force is equal
to the change in kinetic energy of the body.

Potential Energy = energy of position

Gravitational Potential Energy = a type of potential energy which refers to the height
of an object above some reference point

Law of Conservation of Total Energy = the total energy of an isolated system is always
conserved

Conservative Force = if the work done by or against it in moving an object is


independent of the object’s path

Non-conservative Force = depend on path

Total Mechanical Energy = the sum of the kinetic and potential energies

Conservative System = one in which only conservative forces do work

Law of Conservation of Mechanical Energy = in a conservative system, the sum of all


types of kinetic energy and potential energy is constant and equals the total mechanical
energy of the system

Power = rate of doing work

Watt (unit) = the SI power unit

Horsepower = a larger British unit of power

Efficiency = a measure of what you get out for what you put
Important Equations:
Page 64 of 149
Work
W =( FCosθ )d
Where:W = work
F = force
D = distance

Hooke’s Law (Spring Force)


F S=−kx
Where:FS =
k=
x=

Work Done in Stretching or Compressing a Spring (from Equilibrium)


W = 1/2kx2

Kinetic Energy
K = 1/2 mv2
Where:K = kinetic energy
m = mass
v = velocity

Work – Energy Theorem


W = Kf – Ki

Elastic (Spring) Potential Energy


U = 1/2kx2

Gravitational Potential Energy


U = mgh

Conservation of Mechanical Energy


1/2mvf2 + Uf = 1/2mvi2 + Ui

Conservation of Energy
WNC = Ef – Ei = ΔE

Average Power
P = W/t = Fd/t = Fv = FdCosθ/t

Efficiency
ε = Wout/Ein x 100 = Pout/Pin x 100

Example 1: A student holds her psychology textbook, which has a mass of 1.5kg, out of
a second – story dormitory window until her arm is tired, and then she releases it. How

Page 65 of 149
much work is done on the book by the student in simply holding it out the window? How
much work will have been done by the force of gravity during the time in which the book
falls 3.0m?

Given: b . W wg =?
m=1.5 kg
S y =3.0 m Solution:
a . W w =FS=1.5 ( 9.81 ) ( 0 )=0 J
Required:
a . W w =? b . W wg =FS=1.5 ( 9.81 )( 3 ) =44.15 J

Example 2: If the person pushes on the lawn mower with a constant force of 90N at an
angle of 40o to the horizontal, how much work does she do in pushing it a horizontal
distance of 7.50m?

Given: F = 90N
F=90 N
S=7.5m 40o
θ=40 °

Required: 7.5m
a . W w =?

Solution:
a . W w =FS=90 cos 40 ( 7.5 )=517.08 J

Example 3: A 0.75kg block slides with a uniform velocity down a 20o inclined plane.
How much work is done by the force of friction on the block as it slides the total length
of the plane? What is the net work done on the block? What happens if the angle of
incline is adjusted so that the block accelerates down the plane?

Given:
0.75
m=0.75 kg kg
S=1.2m
θ=20° 20o
Required:
a . W wf =?
b . W wt =?
1.2m
Solution:
W = (0.75)(9.81) = 7.36
Wy = 7.36Cos20 = 6.92 N = NF
uk = Tan 20 = 0.36

f = 0.36 (6.92) = 2.49 N

Page 66 of 149
W wf =2.49 ( cos1.220 )=−3.18 J
Wx = 7.36Sin 20 = 2.52 N
1.2
W done Wx =2.52 (
cos 20 )
=3.22 J
W net = Wdone,x + Wf = 3.22 + (- 3.18) = 0.04 J

Example 4: A 0.15kg mass is suspended from a vertical spring and descends a distance
of 4.6cm, after which it hangs at rest. An additional 0.50kg mass is then suspended from
the first. What is the total extension of the spring?

Given:
M1 = 0.15 kg F = kx
X1 = 4.6 cm = 0.046 m -1.47 = k (-0.046 m)
M2 = 0.50 kg k = 31.96 N/m

Required: F = 0.65 (-9.81) = -6.38 N


Xt = ?
F = kx
Solution: -6.38 = 31.96 (XT)
F = W = mg = (0.15) (-9.81) = -1.47 N XT = -0.20 m = -20 cm

Example 5: A shuffleboard player pushes a 0.25kg puck, initially at rest, such that a
constant horizontal force of 6.0N acts on it through a distance of 0.50m. What are the
kinetic energy and the speed of the puck when the force is removed? How much work
would be required to bring the puck to rest?

Given: Solution:
m = 0.25 kg a . W W =FS=6 ( 0.5 )=3 J =K . E .
Vi = 0 m/s
F=6N b . K . E .=0.5 m( V 2f −V 2i )
S = 0.5 m
3=0.5(0.25)(V 2f −02)
3
Required:
a . KE=?
b . v =?
Vf=
√ 0.25(0.5)
=4.90 m/s

c .W w =? c . K . E .=0.5 m(V 2f −V 2i )
K . E .=0.5 ( 0.25 ) ( 02−4.902 )=−3 J

Page 67 of 149
Example 6: A car traveling at 5.0m/s speeds up to 10m/s, with an increase in kinetic
energy that requires work. Then, the speed is increased from 10m/s to 15m/s, requiring
work. How do the amounts of work compare?

Given: Required:
V 1=5 m/s ∆ KE 2
=?
V 2=10 m/s ∆ KE 1
V 3=15 m/ s

Solution:
∆ KE 2 0.5 m(V 23−V 22) 0.5 m(15 2−102 )
= = =1.67
∆ KE 1 0.5 m(V 22−V 21) 0.5 m(10 2−52 )

Example 7: A 0.50kg ball is thrown vertically upward with an initial velocity of 10m/s.
What is the change in the ball’s kinetic energy between the starting point and its
maximum height? What is the change in the ball’s potential energy between its starting
point and the maximum height?

Given:
Vf = 0 m/s
Vf = 0m/s
Vi = 10 m/s
m = 0.5 kg

Required:
a . ∆ KE=?
b . ∆ PE=?
10m/s
Solution:
a . ∆ KE=0.5 m(V 2f −V 2i )
a . ∆ KE=0.5 ( 0.5 ) ( 02−102 )=−25 J

b . V 2f =V 2i +2 gy
02 =102+ 2 (−9.81 ) ( y max ⁡)
−100
y max ⁡= =5.1 m
2(−9.81)

∆ PE=mg(h f −h i)
∆ PE=0.5 ( 9.81 )( 5.1−0 )=25 J

Page 68 of 149
Example 8: A painter on a scaffold drops a 1.50kg can of paint from a height of 6.0m.
What is the kinetic energy of the can when it is at a height of 4.0m? With what speed
will the can hit the ground?

Given:
m = 1.50kg Required:
h = 6.0m a . ∆ KE=? @h=4 m
Vi = 0m/s b . v =?
Solution:
v 2f =v 2i +2 gy
v 2f =02 +2 (−9.81 )(−2 )=6.26 m/s 2

∆ KE =0.5 m ( V 2f −V 2i )
∆ KE =0.5 (1.50 ) ( 6.26 2−02 ) =29.39

Wk =∆ KE+∆ PE
FS=0.5 ( m) ( v f 2−v 2i ) +mg(h f −hi)
0=0.5 ( 1.5 ) ( v f 2−02 ) + 1.5(9.81)(0−6)
1.5 ( 9.81 ) (6)
Vf=
√ 0.5(1.5)
=−10.85 m/ s

Example 9: A 0.30kg mass sliding on a horizontal frictionless surface with a speed of


2.5m/s strikes a light spring that has a spring constant of 3.0 x 10 3N/m. What is the total
mechanical energy of the system? What is the kinetic energy of the mass when the spring
is compressed a distance of 1cm?

Given:
k =3.0 x 103 N /m
m=0.3 kg
v=2.5 m/s
x 1=1 cm=0.01 m

Required:
a . TME=?
b . KE=?

Solution:
a . TME=0.5 m v 2
TME=0.5 ( 0.3 ) ( 2.5 )2=0.94 J

b . KE=TME−0.5 k x 2
KE=0.94−0.5 ( 3.0 x 103 ) ¿

Page 69 of 149
Example 10: A skier with a mass of 80kg starts from rest and skis down a slope from an
elevation of 110m. The speed of the skier at the bottom of the slope is 20m/s. Show that
the system is non-conservative. How much work is done by the non-conservative force
of friction?

Given: V f =20 m/s


m=80 kg h f =0 m
V i=0 m/ s
hi =110 m Required:
W k =?

Solution:
W k =∆ KE+∆ PE
W k =0.5 ( m ) ( v f 2−v 2i ) +mg(hf −hi )
W k =0.5(80)(20 2−02 )+ 80(9.81)(0−110)=−70 328 J

Example 11: A crane hoist lifts a load of 1metric ton a vertical distance of 25m in 9s at a
constant velocity. How much useful work is done by the hoist each second?

Given: t=9 s
m=1 metric ton=1000 kg
h=25 m Required:
P=?

Solution:
W k FS WS ( 1000 ) ( 9.81 ) (25 ) J
P= = = = =27250 =27250 W
t t t 9 s

Example 12: The motors of two vacuum cleaners have net power outputs of 1hp and
0.5hp, respectively. How much work in joules can each motor do in 3min? How long
does it take for each motor to do 97kj of work?

Given: W k =97 KJ=97,000 J


746 J / s
Po 1=1 Hp x =746 J / s
1 Hp Required:
746 J /s a . W k =?
Po 2=0.5 Hpx =373 J /s
1 Hp b . t =?
60 s
t=3 mins x =180 s
1 min

Solution:
Wk
P= ; W k =Pt
t
W k1=746 (180 )=134,280 J W k2 =373 ( 180 )=67,140 J

Page 70 of 149
Wk Wk
P= ; t=
t P
97000 97000
t 1= =130.03 s t 2= =260.05 s
746 373

Example 13: The motor of an electric drill with an efficiency of 80% has a power input
of 60W. How much useful work is done by the drill in a time of 30s?

Given:
% eff =80 %=0.8
Pi=60 W
t=30 s

Required:
W k =?

Solution:
P
eff = o ; Po =eff ( Pi)
Pi
J
Po =eff ( Pi ) =0.8 ( 60 )=48 W =48
s
Wk J
P= ; W k =Pt =48 ( 30 s )=1440 J
t s

Example 14: Determine the potential energy, kinetic energy and total energy of a 1kg
ball initially at rest 50m above the ground and release at a height of 40m, 30m, 20m, 10m
and 0m.

Given:
m=1 kg Required:
V i=0 m/ s PE=?
hi =50 m KE=?
ME=?
Solution:
V 2f =V 2i +2 gy=0+2(9.81)( y )
U =PE=mgh
K=½ mV 2f
ME=KE+ PE
height y Vf2 PE KE ME
50 0 0 490.5 0 490.5
40 10 196.2 392.4 98.1 490.5
30 20 392.4 294.3 196.2 490.5
20 30 588.6 196.2 294.3 490.5
10 40 784.8 98.1 392.4 490.5

Page 71 of 149
0 50 981.0 0 490.5 490.5
Example 15: A block at a bottom of a plane inclined at one angle of 37ْ above the
horizontal is initially at rest. A force,”P”, pulls the block up the plane covering a distance
of 30m with a final velocity of 20m/s. (mass = 1 kg). Determine: a.) PE i, b) PEf, c) KEi,
d) KEf, e) ΔPE, f) ΔKE, g.)TIE, h.)TFE

Given: a . PE i=?
m=1 kg b . PEf =?
θ=37 ° c . KE i=?
V i=0 m/ s d . KE f =?
hi =0 m e . ∆ PE=?
V f =20 m/s f . ∆ KE=?
h f =30 sin 37=18.05 h . TIE=?
i. TFE=?
Required:
P
Solution:

a . PEi=mg hi=1 ( 9.81 ) ( 0 )=0 J 1kg


b . PE f =mg hf =1 ( 9.81 )( 18.05 )=177.11 J 37
o
c . KE i=0.5 mV 2i =0.5 ( 1 ) ( 0 )2=0 J
d . KE f =0.5m V 2f =0.5 ( 1 ) ( 20 )2=200 J
e . ∆ PE =PE f −PE i=177.11−0=177.11 J
f . ∆ KE=KE f −KE i =200−0=200 J
g .TIE=PE i + KE i=0+ 0=0 J
h . TFE=PE f + KE f =177.11+ 200=377.11 J

SIMPLE MECHANICAL MACHINE


A force is needed to move a given load, Effort when the load (W) moves through the
distance (Sw) & the effort (E) also move through the distance (Se).

Relationship

Se
1] IMA= Sw
W
2] AMA= E
3] Wo=W × Sw
4] W i =E× Se
Wo W ( SW )
W / E AMA
5] % eff .= Wi = E( Se) = Se /Sw = IMA

IMA of some Simple Machines

Page 72 of 149
1] Compound wheels & axle
2c
IMA=
b−a

C = radius of the largest wheel


B = radius of the larger wheel
A = radius of the smallest wheel

2] Simple wheel & axle


2 πR R
IMA= =
2 πr r

R = radius of wheel
r = radius of axle

3] Inclined plane
length of plane
IMA=
ht . of plane

4] Pulley
Se
IMA= =1
Sw

5] Pulley system
no . of stands
IMA=
strings

Example 1. A force of 50N is needed to raise a 25kg load to height of 10m using the
pulley system. The load goes up 1m in every 5m of rope pulled from the pulley.
Determine: IMA, AMA, % eff, Work input and work lost

Given: Required:
E = 50N a. IMA
m = 25kg b. AMA
h = 10m c. %Eff
ratio: 1:5 d. Winput
e. Wlost
Solution:
Se 5
a . IMA= = =5
Sw 1

W mg 25× 9.81
b . AMA= = = =4.905
E E 50

4.905
c . % eff = × 100=98.1%
5

Page 73 of 149
d .W i=ESe=50× 50=2500 J

W o =W S W =25 ( 9.81 )( 10)=2452.5 J

e .W L =( 1−eff ) ( W i ) =( 1−0.981 ) (2500)=47.5 J


or
W L =W i−W o=2500−2452.5=47.5 J

Example 2. An inclined plane is 15m long at 3m high. Det. a] IMA b] AMA & c] % eff.
if a 300N force is required to slide 100kg box up the plane.

Given:
L=15 m Required:
h=3 m a. IMA
E=300 N b. AMA
m=100 kg c. % Eff

Solution:
L 15
a . IMA= = =5
H 3

W 100(9.81)
b . AMA= = =3.27
E 300

3.27
c . % eff = (100)=65.4 %
5

Example 3. A wheel has radii of 20cm. & 5cm. if it is a 90% efficient and used to lift a
100kg load to a height of 20m. Solve for: a. E, b. Se, c. Wi, d. Wo, e. Wl

Given:
R=20 cm Required:
r =5 cm a. E
eff =0.9 b. Se
m=100 kg c. Wi
ht=S w =20 m d. Wo
e. Wl
Solution:
R 20
IMA= = =4
r 5

AMA AMA
Eff = ; 0.9= ; AMA=0.9 ( 4 )=3.6
IMA 4

Page 74 of 149
W 100(9.81) 100( 9.81)
a . AMA= ; 3.6= ; E= =272.5 N
E E 3.6

Se Se
b . IMA= =4= ; S e =4 ( 20 )=80 m
Sw 20

c .W i=E S e =272.5(80)=21800 J

d .W o=W S w =100 ( 9.81 ) (20)=19620 J

e .W L =W i−W o=21800−19620=2180 J

Page 75 of 149
Name:
Section:

Solve for net work in each problem.


1.
μ=0.03

W =50 ( 9.81 )=490.5 N =N F F = 250N


m = 50kg
f =490.5 ( 0.03 )=14.72 N

W net =ΣFd= ( 250−14.72 ) (5 )=1176.4 J S = 5m

2. μ = 0.025

W =45( 9.81)=441.45 N
P x =500 cos 40=383.02 N P = 500N
P y =500 sin 40=321.39 N
40o
N F=441.45 – 321.39=120.06 N m = 45kg
f =0.025 9120.06 ¿=3.00 N
S = 4.5m
W net =ΣFd=(383.02 – 3)(4.5)=1710.09 J

3.
μ = 0.02

W =40(9.81)=392.4 N
W x =392.4 sin 10=68.14 N
W y =392.4 cos 10=386.44 N=N F g
B = 300N 40k
S= 4m
f =0.02(386.44)=7.73 N
W net =ΣFd 10o
W net =( 300−68.14−7.73 )( 4 )=896.52 J

4.
μ=0.1 B = 50N
20kg 60o
W =20(9.81)=196.2 N
W x =196.2 sin15=50.78 N
W y =196.2cos 15=189.51 N S = 3m
Bx =50 c 0 s 60=25 N 15
B y =50 sin 60=43.30 N o

Page 76 of 149
N F=189.51+ 43.30=232.81 N
f =0.1(232.81)=23.28

W net =ΣFd=(50.78 – 25 – 23.28)(3)=2.5(3)=7.5 J


5.

Vi = 2m/s Vf = ?

F = 25N
2kg 2kg

S = 4m

W K =∆ PE+∆ KE
FS=( PE f −PE i) +( KE f −KE i)
25 ( 4 )=0.5 ( 2 ) (V ¿ ¿ f 2−22 )¿

100 m
v=
√ 0.5 (2)
+4=10.2
s

6. A 0.5kg. stone is thrown vertically upward with a velocity 30m/s from a point
50m above the ground. Solve for the velocity when it strikes the ground and the height
from the ground when its final velocity is 40m/s. Use energy – work equation.

Given: h=50 m
m=0.5 kg
V i=30 m/ s Required:
V f =40 m/ s a . V f =?
b . h=?

Solution:
a . W K =∆ PE+ ∆ KE

FS=( PE f −PE i) +( KE f −KE i)

0=PE f + KE f −KE i−PEi

KE i + PE i=PE f + KE f

mg hi +0.5 mV 2i=mgh f +0.5 mV f 2


50m
2 2
( 0.5 ) ( 9.81 ) ( 50 )+ 0.5 ( 0.5 ) 30 =( 0.5 ) ( 9.81 ) ( 0 ) +0.5 ( 0.5 ) v f

Page 77 of 149
470.25 m
vf =
√ 0.5(0.5)
=43.37
s

b . W K =∆ PE+ ∆ KE
FS=( PE f −PE i) +( KE f −KE i)

0=PE f + KE f −KE i−PEi


KE i + PE i=PE f + KE f

mg hi +0.5 mV 2i=mgh f +0.5 mV f 2


( 0.5 ) ( 9.81 ) ( 50 )+ 0.5 ( 0.5 ) (30¿¿ 2)=( 0.5 ) ( 9.81 ) ( hf ) +0.5 ( 0.5 ) (40¿¿ 2) ¿ ¿

470.25−400 70.25
hf= = =14.32m
0.5(9.81) 4.905

7. A 1500kg car is coasting down a 30ْo hill at a time when the car’s speed is 2m/s,
the driver applies the brakes. What force (parallel to the road) must be applied by the
brakes of the car is to stop after covering 30m?

Given: S=30 m
θ=30° V f =0 m/s
m=1500 kg
V i=2 m/ s Required
F=?

Solution:
hi hi
Sinθ= ; sin 30= ; h i=30 sin 30=15 m
S 30
30
W K =∆ PE+∆ KE o
FS=mg ( hf −hi ) +0.5 m(V 2f −V 2i )
F ( 30 ) =1500 ( 9.81 ) (0−15)+0.5 ( 1500 ) (0 2−22)

−223,725
F= =7457.5 J
30

8. What power does a body if the body does a 300,000J of work during a time of
30minutes.

Given: Required:
W k =300,000 J P=?
60 s
t=30 min x =¿ 1800 s ¿
1 min Solution:

Page 78 of 149
W 300,000 J
P= P= =166.67 watts
t 1800 s

9. The power developed by a mechanical is 10 HP. How much work was done
during 1 hour?

Given:
746 J /s Required:
P=10 Hp x x 7460 J /s
1 Hp W k =?
t=1 hr=3600 s

Solution:
Wk
P= ; W k =Pt =7460 ( 3600 )=26856000 J
t

10. A machine has an efficiency of 90% and it is used to lift a 500kg load to a height.
of 10m. If its power input is 3HP, determine work output, time it took to operate and
work lost during the process.

Given:
Eff =90 %=0.90 Required:
m=500 kg a . W o =?
h=10 m b . t =?
746 J /s c .W l=?
Pi=3 Hp x =2238 J /s
1 Hp

Solution:
a . W o =mgh=500 ( 9.81 ) (10)=49050 J

Po P
eff = ; 0.90= o ; Po=0.90(2238)=2014.2 watt
Pi 2238

Wo W o 49050
b . P o= ; t= = =24.35 s
t P 2014.2

W i =P i t=3 Hp ( 7461 Hpwatt )( 24.35 s)=54495.3 J


c .W l=W i (1−eff )=54495.3 ( 1−0.90 )=5449.53 J

11. A machine has an eff. of 25% and an input power of 5Hp. Solve for the time it
takes to lift a 1,000 kg load to a height of 30m and the power lost during the process.

Given: m=1000 kg
Eff =25 %=0.25 h=30 m

Page 79 of 149
746 J /s Required:
Pi=5 Hp x =3730 J / s
1 Hp a . t =?
b . W l=?
Solution:
W o =mgh=1000 ( 9.81 ) (30)=294300 J

Po Po
eff = ; 0.25= ; P =0.25(3730)=932.5 watt
Pi 3730 o

Wo W o 294300
a . P o= ; t= = =315.60 s
t P 932.5

b . W l= ( Pi−Po ) t=( 5 ( 746 )−932.5 ) (315.6 )=882 891 J

12. A machine can do 200,000J of work in 2minutes. If the input power is 2.8 HP.
Solve for the efficiency and power lost in Hp during the process.

Given:
W o =200,000 J Required:
60 s a . %eff =?
t=2 min x =¿120 s ¿
1min b . Pl=?
746 J /s
Pi=2.8 Hp x =2088.8 J /s
1 Hp

Solution:
W o 200000
Po = = =1666.67 J / s
t 120

Po 1666.67
a . % eff = = x 100=79.79 %
Pi 2088.8

b . Pl=Pi ( 1−eff )=2.8 ( 1−0.7979 )=0.5659 HP

13.
μ = 0.09

W =40 kg ( 9.81m/ s2 ) =392.4 N g


W X =392.4 sin 10=68.14 N B = 400N 40k
S= 6m
W Y =392.4 cos 10=386.44 N
10o

f =( 386.44 )( 0.09 ) =34.78 N

W net =6 ( 400−68.14−34.78 ) =1782.48 J

Page 80 of 149
14.
μ = 0.1
B = 90N
W =30 kg ( 9.81 )=294.3 N 3 0 kg 45o
W Y =294.3 cos 5=293.18
W X =294.3 sin 5=25.65 N
B X =90 cos 45=63.64 N
S = 4m
BY =90 sin 45=63.64 N 5o
N f =293.18+63.64=356.82 N

f =0.1 ( 356.82 )=35.68 N

W net =(−37.99+35.68 ) 4=−9.24 J

Page 81 of 149
Chapter 6

Momentum and Collisions


Important Terms:

Linear Momentum = the product of mass and velocity

Total Linear Momentum = the vector sum of the moments of the individual particles

Impulse = the change in momentum

Impulse – momentum theorem = the impulse exerted on a body is equal to the change
in the body’s momentum

Conservation of Linear Momentum = No net external force acting on the system

Elastic Collision = the total kinetic energy of all the objects of the system after collision
is the same as their total kinetic energy before the collision.

Inelastic Collision = total kinetic energy is conserved

Completely Inelastic Collision = the objects stick together and hence have the same
velocity after colliding

Center of Mass (CM) = the point at which all of the mass of an object or system may be
considered to be concentrated, for the purposes of linear or translational motion only.

Center of Gravity = the point where all of the weight of an object may be considered to
be concentrated in representing the object as a particle.

Jet propulsion = the application of such jets to the production of motion

Important Equations:

Linear Momentum
p=mv
Where: p = linear momentum
m = mass
v = velocity

Page 82 of 149
Total Linear Momentum of a System
P=∑ i Pi

Newton’s Second Law in Terms of Momentum


Δp
F=
Δt
Where: F = force
T = time

Impulse – Momentum Theorem


J=FΔt= Δp=mv f – mv i

Where: J = impulse

Conditions for an Elastic Collision


Pf =Pi
K f =K i
Where: K = kinetic energy

Conditions for an Inelastic Collision


Pf =Pi
Kf <Ki

Final Velocities in Head - On – Two – Body Elastic Collision


(m – m2 )v 1 i
v1 = 1
(m 1 +m 2)
2m1 v 1 i
v 2=
(m1+ m2 )

Coordinate of the Center of Mass


❑i m i x i
X́ =
M

Example 1: A 100kg football player runs with a velocity of 4.0m/s straight down the
field. A 1.0kg artillery shell leaves the barrel of a gun with a muzzle velocity of 500m/s.
Which has the greater momentum, the football player or the shell?

Given: m
v s=500
m p=100 kg s
m
v p =4.0
s Required:
m s =1kg p p≤¿ p s ?

Page 83 of 149
Solution: ps =mv
p p=mv m m
m m s s (
ps =1 kg 500 =500 kg ∙ =500 N ∙ s )
( )
p p= (100 kg ) 4.0 =400 kg ∙ =400 N ∙ s
s s

p p < ps

Example 2: What is the momentum of a 100kg a shell if its speed is 1500m/s?


Given:
m=100 kg Solution:
v=1500 m/s ρ=mv
ρ=( 100 kg ) (1500 m/s)
Required: ρ=150000 kg ∙m/ s
ρ=?

Example 3: Consider three objects, a typical .22-caliber bullet would have a weight of
about 30 grains muzzle velocity of about 1300ft/s and if 1lb = 7000gr., a ship of about
70,000tons and a speed of about 20knots, if 1knot = 1.15mi/h and a glacier of 1km wide,
10 km long and 250m deep and might have a velocity of 1m/day. Which of the three has
the greatest and least linear momentum and the greatest and the least kinetic energy?

Given:
1 lb 1kg
m 0.22 b=30 gr × × =1.95 ×10−3 kg
7000 gr 2.2 lb
ft 1m
v 0.22b =1300 × =396.34 m/ s
s 3.28 ft

m ship =70000 tons ( 2000


1ton
lbs
)( 1 kg
2.2 lbs )
=63.64 x 10 kg 6

1.15 mi /hr 5280 ft 1m 1 hr


=20 knots ( )( )( 3.28 ft 3600 s )
)(
v ship =10.28 m/s
1 knot 1 mi

1000 kg
m glacier =( 1000 )( 10000 ) ( 250 )
( m 3 )
=2.5× 1012 kg

m
(
v glacier = 1
day )( 124dayhr )( 1hr
3600 s )
=1.16 ×10−6
m
s

Solution:
m
p0.22 b=mv=( 1.95 × 10−3 kg ) 396.34 ( s )
=0.77 kg ∙ m/s

pship =mv=( 63.64 x 106 kg ) 10.28 ( ms )=654.22 ×10 kg ∙ m/s 6

m
kg ) ( 1.16 × 10
s)
p glacier=mv=( 2.5 ×1012 −6 6
=2.90× 10 kg ∙ m/s

Page 84 of 149
KE 0.22 b=0.5 m v 2=0.5 ( 1.95 ×10−3 kg ) (396.34 m/ s)2=153.16 J
m 2
(
KE ship =0.5 m v 2=0.5 ( 63.64 x 106 kg ) 10.28 )
s
=3.36 ×10 9 J

m 2
(
KE glacier =0.5 mv 2=0.5 ( 2.5× 1012 kg ) 11.57 × 10−6 )
s
=167.33 J

Example 4: What is the total momentum of the systems of particles?


a. b.

p2 = 3kg m/s p1 = 2kg m/s

Solution:
m m
PT =P1+ P 2=2+3=5 kg ∙ P1∧2=−8+ 5=−3 kg ∙
s s

m
PT =√( 4)2+(−3)2=5 kg ∙
s
Example 5: A golfer drives a 0.10kg ball from an elevated tee, giving the ball an initial
horizontal speed of 40m/s. The club and the ball are in contact for 1.0milliseconds.
What is the average force exerted by the club on the ball during this time?

Given:
m=0.1 kg Solution:
v=40 m/s J= p
t=1 ×10−3 s Ft =mv
mv 0.1(40)
F= = =4000 N
Required: t 1× 10−3 s
F=?

Example 6: A boy kicks a stationary can with his toe. If he exerts 8lbs. for a time of 0.5s,
how large is impulse applied to the can?

Given:

Page 85 of 149
F=8 ¿
t=0.5 s

Required:
J=?

Solution:
J=Ft =8 ¿( 0.5 s)=4 ¿ s

Example 7: Two masses, m1 = 1kg and m2 = 2kg, are held on either side of a light
compressed spring by a light string joining them. The string is burned and the masses
move apart on the frictionless surface, with m1 having a velocity of 1.8m/s to the left.
What is the velocity of m2?

Given: V 2=?
m 1=1 kg
m 2=2 kg Solution:
m m1 V 1 =m2 V 2
V 1=1.8
s 1(1.8)=2 V

Required:
1.8 m
V 2= =0.9
2 s

Example 8: A ball of 0.3kg mass is thrown toward a wall w/a horizontal velocity of
40m/s. Determine a) impulse exerted in the ball by the wall if the final velocity is 30m/s;
b.) if they were in contact for 1/10 sec. what force was exerted by the wall in the ball?

Given:
m=0.3 kg
V i=40 m/s
V f =30 m/s
1
t= s
10

Required:
a . J =?
b . F=?

Solution: Vi = 40m/s
b . Ft=mV
1
F ( )=m ( V f −V i )
10
Vf = 30m/s

Page 86 of 149
0.3 (30+ 40 )
F= =210 N
0.1

a . J =Ft
J=210 N ( 101 s)=21 N ∙ s

Example 9: A 200g of bullet is fired with a velocity of 500m/s from a 1.5kg rifle. What
is the speed of the recoil of rifle?
Given:

mbullet =0.2 kg Solution


V bullet =500 m/s mbullet V bullet =mriffle V riffle
mriffle =1.5 kg ( 0.2 kg )( 500 m/s )=( 1.5 kg ) V riffle
( 0.2 kg )( 500 m/s )
V riffle =
Required: ( 1.5 kg )
V riffle=?
m
V riffle =66.67 Example 10: A 200g moving at 400m/s hits a bag of sand & comes to rest
s
in 0.0011s. Find the force that stopped the bullet?

Given:
m=0.2 kg
vi =400 m/s
v f =0 m/s
t=0.0011 s

Required:

Solution:
Ft =m ( V f −V i )
( 0.2 kg ) ( 0−400 )
F=
0.0011
F=−72727.27 N

Example 11: A stationary ball is hit by force of 30N for a time of 0.02s. How large is
impulse? What is the momentum after the impulse?

Given:
ViPage 87 of 149
= 40m/s
F=30 N
t=0.02 s

Required:
a . J =?
b . ρ=?

Solution:
a . J =Ft
J= (30 N ) ( 0.02 s )
J=0.6 N ∙ s

b . J = p=0.6 N ∙ s
Example 12: A moving shuffleboard puck has a glancing collision with a stationary one
of the same mass. If friction is negligible, what are the speeds of the pucks after the
collision?

Solution:
Σ ρh=0+→
m1 v 1+ m2 v 2−m1 u1 +m2 u2 =0
V1 = 0.95m/s
m 1 v 1+ m2 v 2=m1 u1 +m 2 u2
v1 =u1+ u2
0.95=u1 cos 50° +u 2 cos 40° → Eqn1

Σ ρv =0+↑
m1 u1 sin 50 °−m2 u2 sin 40 ° V2 = 0m/s
0=
m U1
0=u1 sin50 °−u 2 sin 40°
u2 sin 40 °
u1 = → Eqn2
sin 50° 50
o
Substitute Equation 2 in Equation 1 40
u sin 40 ° o
0.95= 2 (sin50 ° )
cos 50 °+u 2 cos 40 ° U2

sin 40
0.95=u2 (
tan 40
+cos 40 ° )
0.95
u2= =u2=0.62 m/s
sin 40
+cos 40 °
tan 40
m
∴ u1=0.52
s

Page 88 of 149
Example 13: A 1.0kg ball with a speed of 4.5m/s strikes a 2.0kg stationary ball. If the
collision is completely inelastic, what are the speeds of the balls after collision? What
percentage of the initial kinetic energy do they have after the collision? What is the total
momentum after the collision?

Given:
m 1=1 kg Required:
V 1=4.5 m/s a . U 1∧U 2 =?
m 2=2 kg KE f
b. =?
V 2=0 m/ s KE i
e=0 c . pT =?
Solution:
m1 V 1 +m 2 V 2=m1 U 1 +m2 U 2
1 ( 4.5 ) +2(0)=1u1 +2 u2
4.5=u1 +2 u2 → Equation1

u2−u 1 KE f 0.5(m 1 U f 12+ m2 U f 22 )


a . e= b. =
v 1−v 2 KE i 0.5 (m 1 V i 12+ m2 V i 22)
u2−u1
0=
4.5−0 2 2
KE f 0.5 ( [ 1 ( 1.5 ) ]+ [ 2 ( 1.5 ) ] )
u 2−u1 =
0= KE i 0.5 ( [ 1 ( 4.5 )2 ] + [ 2 ( 0 )2 ] )
4.5
∴ u2=u1
4.5=u1 +2 u1 KE f 2.25+4.5 6.75
= ×100= x 100=33.33 %
m KE i 20.25 20.25
u1=1.5 =u2
s
c . pT =m1 U 1+ m2 U 2=1 ( 1.5 ) +2 (1.5 )

m
pT =4.5 kg ∙ Example 14: A 0.30kg object with a speed of 2.0m/s in the positive x
s
direction has a head on collision with a stationary 0.70kg object located at x = 0. What is
the distance separating the objects 2.5s after collision?

Given: V 2=0 m/ s
m 1=0.3 kg
V 1=2 m/s Required:
m 2=0.7 kg d x ∧d y =?

Solution:
m1 V 1 +m 2 V 2=m1 U 1 +m2 U 2
[ 0.3 ( 2 ) +0.7 ( 0 )=0.3 ( u1 ) +0.7 ( u 2 ) ] 10
6=3 u1+ 7 u2

Page 89 of 149
U 2 −U 1
e=1=
V 1 −V 2
U 2−U 1
1=
2−0
2=U 2−U 1
U 2=2+U 1

6=3 U 1+ 7 ( 2+U 1 )
6=3 U 1+ 14+7 U 1
−8=10 U 1
U 1=−0.8 m/s
U 2=1.2 m/s
X1
U 1= ; X 1=U 1 t=0.8 ( 2.5 )=2 m
t
X2
U 2= ; X 2=U 2 t=1.2 ( 2.5 )=3 m
t
X T = X 1+ X 2=2+3=5 m

Example 15: A 7.1kg bowling ball with a speed of 6m/s has a head on collision with a
stationary 1.6kg pin. What is the velocity of each object after the collision? What is the
total momentum after the collision?

Given: V 2=0 m/ s
m 1=7.1 kg
V 1=6 m/ s Required:
m 2=1.6 kg a . U 1∧U 2 =?
b . ρT =?

Solution:
( m – m 2)V 1 ( 7.1−1.6 ) ( 6 )
a . U 1= 1 = =3.79 m/s
(m1 +m 2 ) 8.7
2 m 1 V 1 2 ( 7.1 ) ( 6 )
U 2= = =9.79 m/s
( m1 +m2 ) 8.7

b . ρT =m1 U 1 +m 2 U 2=7.1 (3.79 )+1.6 ( 9.79 )=42.57 kgm/s

Example 16:
40kg @ 25m/s &: 40kg @ 25 m/s

Given: v 2=25 m/s ←


m1=40 kg
v1 =25 m/s → Required:
m 2=40 kg U 1∧U 2=?

Page 90 of 149
Solution:
m 1 v 1+ m2 v 2=m1 u1 +m 2 u2
( 40 )( 25 )+ ( 40 ) (−25 )=40 ( u1 ) +40 ( u 2)
25−25=u 1+u 2
u1=−u2

a. if e = 1 b. If e = 0
u2−u1 u2=u 1
e=
v 1−v 2 −u1=u 1
u2−u 1 2 u1=0
1= U 1=0 m/s=U 2
25+25
50=u2−u1
50=u2−(−u 2) c. if e = 0.8
50=2u2 40=u2−u1
U 2=25 m/ s 40=u2− (−u2 )
U 1=−25 m/ s 40=2 u2
U 2=20 m/ s

U 1=−20 m/ sExample 17: Three masses, 2kg, 3kg and 6kg are located at positions (3.0,
0), (6.0, 0) and (-4.0, 0), respectively, in meters from the origin. Where is the center of
mass of this system?

Given:
m 1=2 kg (3,0)
m 2=3 kg (6,0)
m 3=6 kg (−4,0 )

Required:
x́=?

Solution:
x́ m=❑i mi xi
11 x́=2 ( 3 ) +3 ( 6 ) +6 (−4 )
11 x́=24−24
x́=0

Example 18: A dumbbell has a connecting bar of negligible mass. Find the location of
the center of mass if m1 and m2 are each 5kg and if m1 is 5kg and m2 is 10kg, and if the
distance of dumbbell is 0.20m and 0.90m from the origin.

a. Given: x 2=0.9 m
m1=m2=5 kg
x 1=0.2 m Required:

Page 91 of 149
x́=? m 2=10 kg
x 1=0.2
Solution: x 2=0.9
m x́ =❑i mi xi
10 x́=5 ( 0.2 ) +5 ( 0.9 ) Required:
10 x́=1+4.5 x́=?
5.5
x́= =0.55 m
10 Solution:
m x́ =❑i mi xi
15 x́=5 ( 0.2 ) +10 ( 0.9 )
15 x́=1+9
b. Given:
m 1=5 kg
10
x́= =0.67 m
15

Example 19: A 75.0kg man stands in the far end of a 50kg boat 100m from the shore. If
he walks to the other end of the 6m long boat, how far is the man then from the shore?
Neglect friction; assume that the center of the mass of the boat is at its midpoint.

Given:
m man =75.0 kg
m boat =50 kg
lenght boat =6 m
d=100m

Required:
x=?

Solution:
m x́ =❑i mi xi
125 x́=75 ( 100 ) +50 ( 97 )
125 x́=12350
x́=98.8 m

m x́ =❑i mi xi
125 ( 98.8 )=75 ( x ) +50 ( x+3 )
125 ( 98.8 )=75 x +50 x+150
125 ( 98.8 )−150=125 x

12200
x= =97.6 m
125

Page 92 of 149
Name:
Section:

1.
40kg @ 25m/s &: 30kg 35 m/s

Given: −5−3 u2
m1=40 kg
V 1=25 m/ s →
60=u2− ( 4 )
4 u2 +5+3 u2
m 2=30 kg 60=
4
V 2=35 m/ s ← 240=7 u2+ 5
240−5=7 u2
Required: 235
U 1∧U 2=? U 2= =33.57 m/ s
7
Solution: −5−3 ( 33.57 )
m1 v 1+ m2 v 2=m1 u1 +m2 u2 U 1=
4
40 ( 25 )+30 (−35 )=40 ( u 1) + 30 ( u 2 ) U 1=−26.43 m/ s
−50=40 u1 +30 u2
b. If e = 0
a. if e = 1 u 2−u1
u2−u1 0=
e= 25+ 35
v 1−v 2 0=u2−u1
u2−u 1 u1=u 2
1=
25+35 −5−3 u2
60=u2−u1 0=u2− ( 4 )
0=7 u2 +5

Page 93 of 149
−5 4 u 2+5+ 3u 2
U 2= −0.71m/ s=U 1 48=
7 4
192=7 u 2+5
c. if e = 0.8 187
u 2−u1 U 2= =26.71 m/ s
0.80= 7
25+ 35 −5−3 ( 26.71 )
48=u2−u1 u1=
4
−5−3 u 2
48=u2− ( 4 )
U 1=−21.28 m/ s

2.
40kg @ 35m/s &: 30kg 25 m/s

Given: 60=u2−u1
m1=40 kg 65−3 u2
v1 =35 m/s →
m 2=30 kg
60=u2−
4( )
4 u2−65+ 3u 2
v 2=25 m/s ← 60=
4
240=7 u2−65
Required:
U 1∧U 2=? 305
U 2= =43.57 m/s
7
Solution:
m1 v 1+ m2 v 2=m1 u1 +m2 u2 U 1=u 2−60=43.57−60=−16.43 m/ s
40 ( 35 )+30 (−25 )=40 ( u 1) + 30 ( u 2 )
650=40 u1 +30 u2 b. If e = 0
65−3 u2 u −u
u1= 0= 2 1
4 35−(−25 )
0=u2−u1
a. if e = 1 u1=u 2
u2−u1
e= 65−3 u2
v 1−v 2
u −u
0=u2− ( 4 )
1= 2 1 4 u2−65+3 u2
35− (−25 ) 0=
4

Page 94 of 149
0=7 u2−65

65
U 2= =9.29m/ s=U 1
7

c. if e = 0.8
u 2−u1
0.80=
35+ 25
48=u2−u1
65−3 u2
48=u2− ( 4 )
4 u 2−65+3 u2
48=
4
192=4 u2−65+3u 2
192+65=7u 2

257
U 2= =36.71 m/s
7

U 1=u 2−48=36.71−48=−11.29 m/s

Page 95 of 149
3.
20kg @ 25m/s &: 40kg 5 m/s

Given:
m 1=20 kg Solution:
V 1=25 m/ s → m 1 V 1 +m 2 V 2=m 1 U 1 +m 2 U 2
m 2=40 kg 20 ( 25 ) +40 (−5 )=20 ( u1 ) + 40 ( u 2 )
V 2=5 m/s ← 300=20u 1+ 40 u2
30−4 u 2
Required: u1 = ∨15−2u 2
2
U 1∧U 2=?

a. if e = 1
u −u
e= 2 1
v 1−v 2
u2−u 1
1=
25+5
30=u2−u1
30=u2−( 15−2u 2 )
30=3u 2−15

45
U 2= =15 m/s
3
U 1=u 2−30=15−30=−15 m/s

b. If e = 0
u 2−u1
0=
25 — 5
0=u2−u1
u1=u 2
0=u2−( 15−2 u2 )
0=3 u2−15
15=3u 2

15
U 2= =5 m/s=U 1
3

c. if e = 0.75
u 2−u1
0.75=
25+ 5
22.5=u2−u1
22.5=u2−( 15−2u 2 )
37.5=3u 2

Page 96 of 149
37.5
U 2= =12.5 m/s
3
U 1=u 2−22.5=12.5−22.5=−10 m/s
Chapter 7

Circular Motion
Important Terms:

Angular Displacement = analogous to linear displacement

Radian (rad) = the angle subtending an arc length that is equal to the radius

Average Angular Speed = the angular displacement divided by the total time to travel
the distance

Average Angular Velocity = analogous to linear velocity

Instantaneous Angular Velocity = analogous to instantaneous linear velocity

Tangential Speed = orbital speed

Period = time it takes for an object in circular motion to make one complete revolution or
cycle

Frequency = the number of revolutions or cycles made in a given time

Hertz (Hz) = unit of frequency

Uniform Circular Motion = occurs when an object moves at a constant speed in a


circular path

Centripetal Acceleration = center seeking acceleration

Centripetal Force = net inward force

Average Angular Acceleration = analogous to linear acceleration

Tangential Acceleration = associated with the tangential speed changes and hence
continuously changes direction

Universal Law of Gravitation = a simple relationship for the gravitational interaction


between two particles, or point mass separated by a distance

Universal Gravitational Constant = 6.67 x 10-11Nm2/kg2

Page 97 of 149
Kepler’s First Law (Law of the Orbits) = planets move in elliptical orbits with the Sun
at one of the focal points

Kepler’s Second Law (Law of the Areas) = A line from the Sun to a planet sweeps out
equal areas in equal lengths of time

Kepler’s Third Law (Law of the Periods) = The square of the orbital period of a planet
is directly proportional to the cube of the average distance of the planet from the Sun

Escape Speed = the initial speed needed to escape from the surface of a planet or moon

Important Equations:

Arc Length
θπr
S=
180
Where: S = arc length
r = radius
θ = in degrees

Angular Kinematic Equations


θ=ωt
ωf + ωi
θ=
2
ω=ω i+ αt
θ=ωi t+ 0.5 α t 2
ω f 2=ω i2+ 2αθ
Where: θ = angular displacement
ω = angular velocity
t = time
α = angular acceleration

Tangential and Angular Speeds


v=rω
Where: v = tangential velocity
r = radius
ω = angular acceleration

Angular Speed

ω= =2 π f
T
Where: ω = angular speed
T = period
f = frequency

Page 98 of 149
Frequency and Period
1
f=
T

Centripetal Acceleration
v2 2
a c = =r ω
r

Where: ac = centripetal acceleration


v = tangential velocity
r = radius
ω = angular velocity

Tangential and Angular Accelerations


a t=rα

Where: at = tangential acceleration


r = radius
α = angular acceleration

Centripetal Force and Acceleration


m v2
F c =mac =
r

Where: Fc = centripetal force


m = mass
ac = centripetal acceleration
v =velocity
r = radius

Newton’s Law of Gravitation


Gm 1 m2
F=
r2
G = 6.67 x 10-11Nm2/kg2

Where: F = force
G = gravitational constant
m = mass
r = radius

Acceleration due to Gravity


G ME
a g=
( R E + h)2

Page 99 of 149
Where: a g = acceleration due to gravity
G = Gravitational Constant
ME = mass of the Earth
RE = radius of the Earth
h = altitude

Gravitational Potential Energy of Two Particles


G m1 m2
U=
r

Where: U = potential energy


G = gravitational constant
m = mass
r = radius

Kepler’s Law of Periods


T 2=KR 3

Where: T = period
K=
R = radius

Escape Speed from Earth


v esc =
√2 G M E = 2 g R
rE
√ E

Where: vesc = velocity of escape


G = gravitational constant
ME = mass of the Earth
rE = radius of the Earth

Energy of Orbiting Satellite Orbiting Earth


−GmM E
E=
2r
K = ‫׀‬E‫׀‬

Where: E = energy
G = gravitational constant
M = mass
r = radius
K=

Page 100 of 149


Example 1: A spectator standing at the center of a circular running track observes a
runner start a practice race 256m due east of her own position. The runner runs on the
same track to the finish line, which is located due north of the observer’s position. What
is the distance of the run?

Given: N
r =256 m
θ=90 °

Required:
S=?

Solution: 90o R = 256m


E
56m
θ π r π ( 90 ) (256)
S= = =402.12m
180 180

Example 2: A sailor measures the length of a distant tanker at an angle of 1 ° 9 ' with a
divided circle. He knows from the shipping charts that the tanker is 150m in length.
Approximately how far away is the tanker?

Given:
θ=1° 9 ' S = 150m
S=150m

Required:
r =? R

Solution: θ = 1o9’
1st
θπr 180 S 180 (150)
S= ; r= = S = 75m
180 πθ π (1 ° 9 ' )

180(150)
r= =7473.36 m y
π (1° 9' )

2nd θ = 0o34.5’
x x
Tanθ= ; y=
y Tanθ

Page 101 of 149


75
y= =7473.11 m
tan 0° 34.5 '

Example 3: An amusement park merry - go - around at its constant operational speed


makes one complete rotation in 45s. Two children are on horses, one at 3.0m from the
center of the ride and the other farther out, 6.0m from the center. What are the angular
speed and the tangential speed of each child?

Given:
t=45 s
r 1=3 m
r 2=6 m

Required: R2 = 6m
a . ω=?
b . V 1=V 2=?
R1 = 3m
Solution:
2π 2π
ω= = =0.14 rad /s
t 45 s

V 1=ω r 1=0.14 (3)=0.42 m/s

V 2=ω r 2=0.14 (6)=0.84 m/s

Example 4: A compact disc rotates in a player at a constant speed of 200rpm. What are
the CD’s frequency and period of revolution?

Given:
200 rev 2 πrad 1 min
ω= × × =20.93 rad /s
min 1 rev 60 s

Required:
a . f =?
b . T =?

Solution:
2π 2π 2π
a . ω= ; t= = =0.30 s
t ω 20.93

1 1
b.f = = =3.33 hz
T 0.30 s

Page 102 of 149


Example 5: A space station is in a circular orbit about the Earth at an altitude of 5 ×102
km. If the station makes one revolution every 95min, what are its orbital speed and
centripetal acceleration?

Given:
h=5.0 ×102 km=5.0 ×105 m
60 s Solution:
t=95 min × =5700 s
min r T =6.4 × 106 m+5.0 ×10 5=6.9 ×106 m
r E=6.4 ×106 m
6
2 πrn 2 π ( 6.9 ×10 m ) (1)
Required: a.VT= = =7605.96 m/ s
t 5700 s
a . V T =?
b . ac =?
v 2 7605.962 2
b . ac = = 6
=8.38 m/s Example 6: A laboratory centrifuge operates at a
r 6.9 ×10
rotational speed of 12000 rpm. What is the magnitude of the centripetal acceleration of
the red blood cell at a radial distance of 8.00cm from the centrifuge’s axis of rotation?
How does this acceleration compare to g?

Given:
12000rev 2 πrad 1min
ω= × × =1256.64 rad /s
min 1rev 60 s
R=8 cm=0.08 m

Required:
ac
=?
g

Solution:
2
v 2 (ωr)
a c= = =rω2 =0.08(1256.64)2=126,331.53 m/s 2
r r

ac 126,331.53
= =12,877.83
g 9.81

Example 7: A car approaches a level, circular curve with a radius of 45.0m. If the
concrete pavement is dry, what is the maximum speed at which the car can negotiate the
curve at a constant speed?

Given: R=45.0 m

Page 103 of 149


F s =F c
max

Required: m v2
V =? μs m g=
R
2
v =μs gR
Solution: v=√ μ s gR=√ 1.2 ( 9.81 )( 45 )=23.02 m/s
Example 8: Suppose that two masses, m 1 = 2.5kg and m2 = 3.5kg, respectively, are
connected by light strings and are in uniform circular motion on a horizontal frictionless
surface where r1 =1.0m and r2 = 1.3m. The forces acting on the masses are T2 = 2.9N and
T1 = 4.5N. Find the centripetal accelerations and the magnitudes of the tangential
velocities of the masses.

Given: T 1=4.5 N
m 1=2.5 kg T 2=2.9 N
m 2=3.5 kg
r 1=1 m Required:
r 2=1.3 m a . V 1=V 2=?
b . a1=a2=?

Solution:
Σ F=ma
T 2=m 2 a 2 F net=T 1−T 2=m 1 a1
T 2.9 N T −T 2 4.5−2.9 1.6 N
a 2= 2 = a 1= 1 = =
m2 3.5 kg m1 2.5 2.5 kg
2
a 2=0.83 m/s
a 1=0.64 m/s 2
V2
a c = ; V =√ ac r
r
V 2= √a c2 r 2=√ 0.83(1.3)=1.04 m/s V 1= √a c1 r 21=√ 0.64(1)=0.8m/ s

Example 9: A compact disc accelerates uniformly from rest to its operational speed of
500rpm in 3.5s. What is the angular acceleration of the CD during this time, after this
time and if the CD comes uniformly to a stop in 4.5s, what is its angular acceleration?

Given: a . α 1=?
w i=0 m/ s b . α 2=?
500rev 2 πrad 1 min c . α 3=?
ω= × ×
min 1rev 60 s
2
ω=52.36 rad / s
t 1=3.5 s Solution:
t 2=∞ ω f =ωi +αt
t 3=4.5 s ω f −ωi
α=
t
Required:

Page 104 of 149


52.33−0 0−0
a . α 1= =14.95 rad /s 2 b . α 2= =0 rad /s 2
3.5 s ∞

0−52.38
c . α 3= =−11.63 rad / s2
4.5

Example 10: A microwave oven has a rotating plate 30cm in diameter for even cooking.
From rest, the plate accelerates at a uniform rate of 0.87rad/s 2 for 0.50s before reaching
its constant operational speed. How many revolutions does the plate make before
reaching its operational speed? What are the constant angular speed of the plate and the
tangential speed of the rim?

Given: ω i=0 m/s


d=30 cm
r =0.15 m Required:
∝=0.87 rad /s2 a . θ=?
t=0.5 s b . ω f =?
c .V f =?

Solution:
v f 2=v i2+2 ax
1
x=v i t + a t 2
2

a . θ=ωi t+0.5 αt =0.5 ( 0.87 ) (0.5)2


360 °
θ=0.11 rad × =6.30 °
2 πrad

b . ω f =ωi +αt=0.87 ( 0.5 )=0.44 rad /s

c .V =ωr =0.44 ( 0.15 ) =0.07 m/s

Example 11: Estimate the magnitude of the mutual gravitational force between the Earth
and the Moon.

Given:
M E =6.0 ×1024
M M =7.4 ×1022
r EM =3.8 ×108

Required:
F=?

Solution:
Page 105 of 149
Nm2 (

F=
Gm1 m2 G M E M M
= =
( 6.67 ×10−11
kg 2 )6.0 ×10 24 kg ) ( 7.4 ×1022 kg )

r2 2 2
( r EM ) ( 3.8× 108 m )
F=2.05 ×1020 N

Example 12: Two 50kilogram satellites are in circular orbits about the Earth at altitudes
of 1000km and 36000km respectively. What is the difference in the gravitational
potential energies of the two satellites in their respective orbits?

Given: M E =6.0 ×1024


M 1=M 2=50 kg R E=6.4 ×106 m
r 1=1 ×10 6 m
r 2=36 × 106 m Required:
GPE=?

Solution:
M2 M1
GPE=−G M E
[ −
RE + h2 R E + h1 ]
50 50
GPE=−6.67 ×10−11 (6.0 ×10 24)
9
[ −
6.4 ×10 +1 ×10 6.4 × 106 +36 ×106
6 6 ]
GPE=2.23 ×10 J

Example 13: What is the total gravitational potential energy of the three masses shown?

Given:
m3 = 2.0kg (0,4.0m)

m1 =1.0kg (0,0) m2 = 2.0kg(3.0m,0)

Required:
U T =?

Solution:
G m1 m2 −6.67 ×10−11 ( 1 ) (2)
U 1,2 = = =−4.45 ×10−11 J
r 3

Page 106 of 149


G m 1 m 2 −6.67 ×10−11 ( 1 ) (2)
U 1,3 = = =−3.34 ×10−11 J
r 4

G m1 m2 −6.67 ×10−11 ( 2 ) (2)


U 2,3 = = =−5.34 × 10−11 J
r 5

U T =U 1,2 +U 1,3 +U 2,3=(−4.45 ×10−11 ) + (−3.34 × 10−11 ) +(−5.34 ×10−11 )


U T =−1.31 ×10−10

Example 14: What is the escape speed at the Earth’s surface?

Given:
R E=6.4 x 106 m

Required:
v=?

Solution:
v=√ 2 g R E= √2 ( 9.81 ) (6.4 × 106)=11,705.71 m/s

Page 107 of 149


Chapter 8

Rotational Motion and Equilibrium


Important Terms:

Rigid Body = an object or system of particles in which the distances between particles
are fixed and remain constant

Translational Motion = every particle has the same instantaneous velocity which means
there is no rotation

Rotational Motion = particles of an object have the same instantaneous angular velocity
and travel in circles about the axis of rotation.

Instantaneous Axis of Rotation = a rolling object rotates about this axis through which
its point of contact with the surface or along its line of contact with the surface.

Moment Arm (lever arm) = the perpendicular distance between the point and the force.

Torque = the product of the force and the lever arm

Translational Equilibrium = the net force on a body is zero

Concurrent Forces = forces with lines of action through the same point

Rotational Equilibrium = the sum of the torques acting on an object is zero

Mechanical Equilibrium = a body is said to be in this state if both the translational and
rotational equilibrium are satisfied.

Static Equilibrium = the condition that exist when a rigid body remains at rest

Center of Gravity = the point at which all the weight of an object may be considered to
be concentrated in representing it as a particle
Page 108 of 149
Stable Equilibrium = any small displacement results in a restoring force or torque,
which tends to return the object to its original equilibrium position

Unstable Equilibrium = any small displacement from equilibrium results in a force and
a torque that tends to rotate the object farther away from its equilibrium position

Moment of Inertia = the summation of the product of the mass and the square of the
distance from the center

Parallel Axis of Theorem = the moment of inertia about such a parallel axis
Rotational Work = force acting tangentially along an arc length

Rotational Power = the rotational analogue of power

Rotational Kinetic Energy = the rotational analogue of kinetic energy

Angular Momentum = the product of a moment arm and a linear momentum

Conservation of Angular Momentum = in the absence of an external, unbalanced


torque, the total vector angular momentum of a system is conserved or remains constant

Important Equations:

Condition for Rolling without Slipping


vCM =rω
( s=rθ∨aCM =r α )

Where: VCM = velocity of the center of mass


r = radius
ω = angular velocity
s = arc length
θ = angular displacement
aCM = acceleration of the center of mass
α = angular acceleration

Torque
¿r F

Where:  = torque
r = perpendicular distance
F = force

Conditions for Translational and Rotational Mechanical Equilibria


F i=0
i =0

Page 109 of 149


Torque on a Particle
¿ r F=F r=rma=m r 2 α

Moment of Inertia
I =mi r i2

Rotational Form of Newton’s Second Law


¿Iα

Parallel Axis Theorem


I =I CM + M d 2

Where: I = moment of Inertia


ICM = moment of inertia of the center of mass
M = mass
d = distance from the neutral axis

Rotational Work
W =θ

Rotational Power
P=ω

Work Energy Theorem


1 1
W =ΔK = I ω f 2 – ωi2
2 2

Rotational Kinetic Energy


1 2
K= I ω
2

Kinetic Energy of a Rolling Object


1 1
K= I CM 2= M v 2CM
2 2

Angular Momentum of a Particle in Circular Motion


L=r p=mr v=mr 2 ω

Angular Momentum of Rigid Body


L=Iω

Torque in Terms of Angular Momentum

Page 110 of 149


ΔL
¿
Δt
Conservation of Angular Momentum
Iω=I i ω i

Example 1: A cylinder with a radius of 12cm rolls with an instantaneous angular speed
of 0.75rad/sec down the inclined plane. If the center of mass of the cylinder travels at the
speed of 0.10m/s at this time, does the cylinder roll without slipping? The cylinder goes
onto a level surface and rolls without slipping with the same speed of the center of mass
as in the first. Assuming this speed remains constant for 2.0s, through what angle does
the cylinder turn during this time?

Given: t=2 s
r =12cm=0.12m
ω=0.75 rad / s Required:
V cm =0.10 rad /s a . V cm ∨V =?
b . θ=?

Solution:
a . V =rω=( 0.12 )( 0.75)=0.09 m/s
0.09m/s < 0.10m/s: the cylinder slips

v 0.10 m/s
ω= = =0.83 rad /s
r 0.12 m

b . θ=ωt
θ=0.83 rad / s ( 2.0 sec. )=1.66 rad

Example 2: In our bodies, torques are produced by the contraction of our muscles causes
some bones to rotate at joints. With the axis of rotation through the elbow joint and the
muscles attached 4.0cm from the joint, what are the magnitudes of the muscle torques for
cases from a 120o and 90o if the muscle exerts a force of 600N?

Given:
d=4 cm=0.04 m
F=600 N
θ1=120 °
θ2=90 °

Page 111 of 149


Required:
τ =?

Solution:
a . τ =F d=600 ( 0.04 ) cos 30=20.78 Nm

b . τ =F d=600 ( 0.04 m ) cos 0=24 Nm

Example 3: Three masses are suspended from a meter stick. How much mass must be
suspended on the right side for the system to be in static equilibrium?

0 20cm 50cm 85cm

25g 75g M =?

Given:
m 1=25 g Required:
d 1=0 cm m 3=?
m 2=75 g
d 2=20 cm Solution:
d 3=85 cm Σ τ=0+cw
center of gravity =50 cm −25 ( 50 )−75 ( 30 ) + x ( 35 )=0

25 ( 50 )+ 75 (30 )
x= =100 g
35
Example 4: A ladder with a mass of 15kg rests against a smooth wall. A painter who has
a mass of 78kg stands on the ladder. What frictional force must act on the bottom of the
ladder to keep it from standing?

Given: R Wp = 78kg
m ladder =15 kg 1.6m
m painter=78 kg
Wl = 15kg
Required: 1.0m
f =?
5.6m
Solution:
Page 112 of 149

f
Σ F h=0+→
R−f =0
R=f

Σ τ b=0+cw
−w l ( 1 ) −w p ( 1.6 ) + R ( 5.6 ) =0
−15 ( 9.81 ) (1 )−78 ( 9.81 )( 1.6 )+5.6 R=0
15 ( 9.81 ) +78 ( 9.81 ) (1.6)
R=
5.6
R=244.90 N

F=R=244.90 N

Example 5: Uniform, identical bricks 20cm long are stacked so that 4.0cm of each brick
extends beyond the brick beneath. How many bricks can be stacked in this way before
the stack falls over?

Given:
l=20 cm Required:
extension=4 cm n=?
Solution:
Σm x́=m i x i
2bricks
m(x 1+ x 0 )
x́=
2m
0+ 4
x́= =2 cm
2

3bricks
m(x 1+ x 2 + x 3)
x́=
3m
0+ 4+8
x́= =4 cm
3

4bricks
m(x 1+ x 2 + x 3+ x 4)
x́=
4m
0+ 4+8+12
x́= =6 cm
4

5bricks
m(x 1+ x 2 + x 3+ x 4+ x5 )
x́=
5m
0+ 4+8+12+16
x́= =8 cm
5
Page 113 of 149
6bricks
m(x 1+ x 2 + x 3+ x 4+ x5 + x 6 )
x́=
6m
0+ 4+8+12+16 +20
x́= =10 cm
6

7bricks
m(x 1+ x 2 + x 3+ x 4+ x5 + x 6 +x )
x́= 5

7m
0+ 4+8+12+16 +20+24
x́= =12 cm
7

12cm exceeds the half the length of the brick at the bottom
Ans: 6bricks
Example 6: Solve for the moments at each point. Each square has a 2m x 2m dimension.

500N; 3:4
A

200N
B D

100N

d= √ 3 2+ 42 =√ 9+16= √25=5

Fx x Fx 500( 4)
= ; F x= = =400 N
F d d 5

Fy y Fy 500 (3)
= ; F y= = =300 N
F d d 5

M A =−300 ( 4 )−200 ( 4 )+ 100 ( 4 )=−1600 Nm

Page 114 of 149


M B =400 ( 4 )=1600 Nm
M C =400 (6 ) +200(2)=2800 Nm
M D =300 ( 4 ) +400 ( 4 )−100 (4)=2400 Nm
M E =300 ( 6 ) + 400 ( 10 )−200 ( 6 )−100 (6)=6400 Nm

Example 7: Find the moment of inertia about the axis indicated for each of the one-
dimensional dumbbell. Consider the mass of the connecting bar to be negligible.

Given:
a. m1 = m2 = 30kg, x1 = x2 = 0.50m
b. m1 = 40kg, m2 = 10kg, x1 = x2 = 0.50m
c. m1 = m2 = 30kg, x1 = x2 = 1.50m
d. m1 = m2 = 30kg, x1 = 0m & x2 = 3.0m
e. m1 = 40kg, m2 = 10kg, x1 = 0m & x2 = 3. 0m

Required:
I=?

Solution:
a. I =m i r i2=30 ( 0.50 )2+ 30¿
b. I =m i r i2=40 ( 0.50 )2 +10 ¿
c. I =m i r i2=2¿
d. I =m i r i2=30 ( 0 )2+ 30 ¿
e. I =m i r i2=40 ( 0 )2 +10 ¿

Example 8: A student opens a 12kilogram door by applying a constant force of 40N at a


perpendicular distance of 0.90m from the hinges. If the door is 2.0m in height and 1.0m
wide, what is the magnitude of the angular acceleration? Assume that the door rotates
freely on its hinges.

Given: h=2.0 m
mdoor =12 kg w=1.0 m
F=40 N
d=0.90 m Required:
Page 115 of 149
α =? 1
I = ( 12 kg ) ( 1.0 m )2
3
Solution: I =4 kg m 2
τ =F d
τ =40 N ( 0.9 m ) τ 36 Nm
∞= =
τ =36 Nm I 4 kg m 2
∞=9 rad /s 2
1
I = m L2
3

Example 10: A solid, rigid spherical ball of mass M and radius R is released at the top of
a hard surfaced inclined plane. The ball rolls without slipping, with only static friction
between it and the plane. What is the acceleration of the ball’s center of mass?

Given: 2
f s= ma
m=m 5
r =r
Σ F=ma
Required: mgSinθ−f s=ma
a cm=? 2
m gSinθ− ma=ma
5
Solution: 7
gSinθ= a
τ =R f s=I ∞ 5
R f s= ( 52 m R )( aR )
2

5
a= gSinθ
7
Example 11: A uniform, solid 1.0kg cylinder rolls without slipping at a speed of 1.8m/s
on a flat surface. What is the total kinetic energy of the cylinder?

Given: Solution:
m=1.0 kg KE=0.5 I ω2 +0.5 m v 2
v=1.8 m/s v 2
KE=0.5 ( 0.5 m R2 ) ( )
R
+0.5 m v 2
Required: KE=0.25 m v +0.5 m v 2
2
KE T =?
KE=0.75 m v 2
KE=0.75 (1)(1.8)2
KE=2.40 J

Page 116 of 149


Example 12: A uniform cylindrical hoop is released from rest at a height of 0.25m near
the top of an inclined plane. If the cylinder rolls down the plane without slipping and
there is no energy loss due to friction, what is the linear speed of the cylinder’s center of
mass at the bottom of the incline?

Given:
h=0.25 m

Required:
v=?

Solution:
E=Eo
mg hi=0.5 m v 2 +0.5 I ω 2
2
v
mg hi=0.5 m v 2 +0.5 ( m R 2 ) ( )
R
mg hi=0.5 m v 2 +0.5 m v 2
m g h i=m v 2

Page 117 of 149


1.57 m
v=√ g hi=√ ( 9.81 ) ( 0.25 )= Example 13: A small ball at the end of a string that
s
passes through a tube is swung in a circle. When the string is pulled down through the
tube, the angular speed of the ball increases, is this caused by a torque due to the pulling
force? If the ball is initially swung at a speed of 2.8m/s in a circle with a radius of 0.30m,
what will be its tangential speed if the string is pulled down far enough to reduce the
radius of the circle to 0.15m?

Given:
v=2.8 m/s
r 1=0.30 m
r 2=0.150 m

Required:
v 2=?

Solution:
I i ω i=I f ω f
( m R2 ) v =( m R2 ) v
( ) ( )
R R
v1 R1=v 2 R2
v 1 R1 ( 2.8 ) ( 0.3 ) m
v 2= = =5.60
R2 0.15 s

Page 118 of 149


Name:
Section:

1. A uniform bar 5m long weighs 200N & is supported at its end by 2 walls. Find the
reaction of the walls against the beam when a man weighing 750N stands on the beam at
a distance 2m from l end.
200N 750N
Given:
l=5 m 2m
2.5m
W b =200 N
W m =750 N

Required: 5m
R A =? RA RB
R B=?

Solution:
Σ τ A =0+cw Σ F V =0+↑
200 ( 2.5 ) +750 ( 3 )−R B (5 )=0 R A + R B −200−750=0
500+2250 R A + 550−200−750=0
R B= =550 N
5 R A =400 N
2. Solve for the moments at each point. Each square has a 2m x 2m dimension.

1300N; 5:12
A

500N
B D

700N
m A =−500 ( 4 )−500 ( 4 )+ 700 ( 4 ) =−1200 Nm
m B =1200( 4)=4800 Nm
mC =1200 ( 6 ) +500 ( 2 )=8200 Nm Page 119 of 149
m D =500 ( 4 ) +1200 ( 4 )−700 ( 4 )=4000 Nm
mE =500 ( 6 ) +1200 ( 10 ) +500 ( 6 )−700 (6)=13800 Nm
Chapter 9

Solids and Fluids


Important Terms:

Fluid – Substances that can flow, liquids and gases qualify

Stress – A measure of the force causing a deformation, the applied force per unit cross
sectional area

Strain – A relative measure of the deformation a stress causes.

Elastic Modulus – The constant of proportionality which depends on the nature of the
material

Elastic Limit – A critical point where the strain begins to increase more rapidly

Young’s Modulus – The elastic modulus for a tension or a compression

Shear Modulus – Sometimes called the modulus of rigidity, the elastic modulus for
shear

Bulk Modulus – The elastic modulus for volume

Compressibility – the reciprocal of bulk modulus

Pressure – force per unit area

Pascal (Pa) – SI unit for pressure which is equivalent to N/m2

Atmosphere (atm) - The average atmospheric pressure at sea level is sometimes used as
a unit

Pascal’s Principle – Pressure applied to an enclosed fluid is transmitted undiminished to


every point in the fluid and to the walls of the container.

Absolute pressure – the sum of the atmospheric pressure and the gauge pressure.

Gauge pressure – The pressure measured by gauges which are above the atmospheric
pressure.

Page 120 of 149


Buoyant force – The upward force resulting from an object being wholly or partially
immersed in a fluid.

Archimedes’ principle – A body immersed wholly or partially in a fluid is buoyed up by


a force equal in magnitude to the weight of the volume of the fluid it displaces.

Specific gravity – The ratio of the density of the substance to the density of the water

Ideal Fluid – is steady, irrotational, nonviscous and incompressible

Streamlines – Every particle moves along the same path as particles that passed by
earlier.

Equation of continuity – If there are no losses of fluid within a uniform tube, the mass
of fluid flowing into the tube in a given time must be equal to the mass flowing out of the
tube in the same time.

Flow rate equation – A1V1 = A2V2

Bernoulli’s Equation - V21/2g + P1/ +z1 = V22/2g + P2/ +z2

Surface tension – The inward pull on the surface molecules causes the surface of the
liquid to contract and to resist being stretched or broken.

Viscosity – All real fluids have an internal resistance to flow

Poiseuille’s law – for viscous flow

Important Equations:
F
σ=
A

Where: σ = stress
F = force
A = area

∆ L L f – Li
ε= =
Li Li

Where: ∆L = change in length


Lf = final length
Li = initial length
ε = strain

Page 121 of 149


Young’s Modulus
stress σ F/A
E= = =
strain ε ∆ L/ Li
Shear Modulus
F / A F/ A
ν= =
x /h ε

Where: x = relative displacement


h = distance

Bulk Modulus
F/ A − Δp
B= =
−ΔV /V i −ΔV /V i

Where: p = pressure
V = volume
Vi = initial volume

Compressibility
1
k=
B

Pressure
F
P=
A

Where: F = force perpendicular to the surface


A = area

Density
m
ρ=
Vol

Pressure – Depth Equation (for incompressible fluid at constant density)


P = Pa + ρgh

Where:Pa = atmospheric pressure


ρ = density
g = gravity
h = depth
P = absolute pressure

P = Pa + Pg

Page 122 of 149


Where: Pg = gauge pressure

Archimedes’ Principle
Fb = mg = ρgV

Where: Fb = buoyant force


m = mass
g = gravity
ρ = density
V = volume

ρf
F b=( )ρo
Wo

Where: ρf = density of fluid displaced


ρo = density of the object
Wo = weight of object

Equation of Continuity
ρ1A1v1 = ρ2A2v2 = Q
Where: Q = discharge
ρ = density
A = area
v = velocity

Bernouilli’ Energy Equation


E = V2/2g + P/ +z

Energy equation at two points


V21/2g + P1/ +z1 = V22/2g + P2/ +z2
Where: E = energy
V = velocity
g = gravity
P = pressure
 = unit weight of fluid being considered
= unit weight of water is 9810KN/m3
Z = elevation head
V2/2g = velocity head
P/ = pressure head

Fluid Friction
Express as head loss

Page 123 of 149


Darcy Weisbach Equation
hf =fLV2/2gD

Where: f = friction coefficient


L = length
V = velocity
g = gravity
D = diameter

English System
hf =0.0252fLQ2/D5

Metric System
hf =0.0826fLQ2/D5

Manning’s Formula
English System
hf =4.65n2LQ2/D16/3

Metric system
hf =10.294n2LQ2/D16/3

Poiseuille’s Law
Q = ¶r4Δp/8ηL

Where: Q = discharge
r = radius
P = pressure
η = viscosity
L = length

Example 1: The femur (upper leg bone) is the longest and strongest bone of the body.
Taking a typical femur to be approximately circular with a radius of 2.0cm, how much
force would be required to extend the bone by 0.010%?

Given:
r = 2.0cm =0.02 m
ε = 0.010 % = 0.0001
E = 1.50 x 1010 N/m2
Required:
F=?

Page 124 of 149


F ∆L
σ=
A
=E
Lo[ ]
∆L
F= AE =( 1.5 x 1010 ) ¿
Lo
Example 2: By how much should the pressure on a liter of water be changed to compress
it by 0.10%?

Given:
1000 cm3 ( 1 m )3
Vol=1 L x x 3
=0.001 m3
1L ( 100 cm )
∆V
=0.10 %=0.001
Vo
Β = 2.2 x 109 N/m2

Required:
P=?

Solution:
∆V
3
=0.001=1 c m3 ¿
1000 c m
9
β ∆ V 2.2 X 1 0 ( 0.000001)
∆ ρ= = =2.2 X 10 6 N /m2
Vo 0.001

Example 3: What is the total pressure on the back of a scuba diver in a lake at a depth of
8.00m? What is the force on the diver’s back due to the water alone; taking the surface of
the back to be a rectangle 60.0cm by 50.0cm?

Given: Pa=1.01 x 105 N /m2


d = 8m
w = 60cm = 0.6m Required:
l = 50cm = 0.5m a . ρ=?
ρ H 20=1000 kg/m3 b . F=?

Solution:
A=0.6 x 0.5=0.30 m 2

1000 kg m
a . ρ= ρa + ρgh=1.0 x 105 +
5
m 2

2
(
9.81 2 ( 8 m )
s )
ρ=1.0 x 10 +78,480=179,480 N / m =179480 Pa=179.48 KPa

b. P = F/A
F=PA=78480 N /m 2 ( 0.30 m 2) =23.544 KN

Page 125 of 149


Example 4: A garage lift has input and lift (output) pistons with diameters of 10cm and
30cm, respectively. The lift is used t hold up a car with a weight of 1.4 x 10 4N. What is
the force on the input piston? What pressure is applied to the input piston?

Given:
di = 10 cm = 0.1 m
do = 30 cm = 0.3 m
Fo = 1.4 x 104 N

Required:
a. Fi = ?
b. P = ?

Solution:
a. Po = Pi
F o Fi Ai F o
= ; Fi = =π ¿ ¿ ¿
A0 Ai A0

1555.56
b . P=
π ¿¿¿

Example 5: If a standard atmosphere supports a column height of exactly 76 cm of


mercury, what is the standard atmospheric pressure in pascals? The density of mercury is
13.5951 x 103kg/m3.

Given:
hHg = 76 cm = 0.76m
PHg = 13.5951 x 103 kg/m3
g = 9.80665

Required:
ρa =?

Solution:
103 kg 9.80665 m
(
Pa= ρHg gh= 13.5951 x
m3 )(
s2 )
( 0.76 m )
Pa=101325.0144 Pa=101.325 KPa

Page 126 of 149


Example 6: Consider a hospital patient who receives an IV under gravity flow. If the
blood gauge pressure in the vein is 20.0mm Hg, above what height should the bottle be
placed for the IV to function properly?

Given:
133 Pa
P=20 mmHg x =2660 Pa
1 mm Hg
pblood = 1.05 x 103 kg/m3

Required:
h=?

Solution:
P= ρgh

P 2660 N /m2
h= = =0.26 m=26 cm
ρg 3 kg m
( )
1.05 X 10 3 (9.81 2 )
m s

Example 7: What is the buoyant force in air on a spherical helium balloon with a radius
of 30cm if ρair = 1.29kg/m3?

Given:
r = 30 cm = 0.3 m
p = 1.29 kg/m3

Required:
Fb = ?

Solution:
4
Vol= π ¿
3

kg m
F b=mg= ρvg= 1.29
( m 3 ) ( )
(0.11 m3 ) 9.81 2 =1.39 N
s

Page 127 of 149


Example 8: A uniform solid cube material 10cm on each side has a mass of 700g. Will
the cube float in water? If so, how much of its volume will be submerged?

Given:
s = 10 cm = 0.1 m
m = 700g = 0.7kg

Required:
%v = ?

Solution:
3 3
vol=( 0.1 ) =0.001m

m 0.700 kg
ρcalc= = =700 kg/m 3
vol 0.001 m 3

1000 kg 700 kg
a . ρ water = 3
> ρ cube= ; yes it will float
m m3

kg
700
Vcube ρcube m3
b. = = =70 % of the cube is submerged
Vwater ρwater kg
1000 3
m

Example 9: High cholesterol in blood can cause fatty deposits called plaques to form on
the walls of blood vessels. Suppose a plaque reduces the effective radius of an artery by
25%. How does the partial blockage affect the speed of blood through the artery?

Given:
Reduction= 25%

Required:
V=?

Solution:
Q1 = Q2
V 1 A 1=V 2 A 2
V 1 π d 21 V 2 π d 22
=
4 4

Page 128 of 149


d 2=(0.75)d 1

V 1 d 21
V 2= 2
=1.78 V 1
( 0.75 d 1 )

Example: 10

Q = 2cms

A B C
D = 1m D = 0.75m D = 0.5m

What are the velocities at points A, B, and C?

Given:
Q=2cms 4 Q 4(2)
V 1= = =2.55 m/ s
π d 2 π ( 1 )2
Required:
V =? 4Q 4(2)
V 2= = =4.53 m/s
π d π ( 0.75 )2
2
Q = VA
Vπ d 2
Q= 4Q 4 (2)
4 =V 3= =10.19m/ s
π d π ( 0.50 )2
2

Example 11: Water escapes through a small hole in a tank. What is the initial flow rate
of the water out of the tank?

V 21 P 1 V 22 P2
+ + z 1= + + z 2
2g γ 2g γ
V1 V2
− =z 2−z 1
2g 2g
V 21−V 22 =2 g ( z 2−z 1 )
Q=Q 2
V 1 d 21=V 2 d 21
V d2
v 2= 1 2 1
d2

Page 129 of 149


2
v 1 d 21
2
V −
1
( ) d 22
=2 g (z ¿ ¿ 2−z1 ) ¿

d 14
2
v 1−
1
( ) d 24
=2 g( z 2−z 1 )

2 g (z 2−z 1) d 24
v1 =
√( d 24−d 14 )
Note: Pressure head is zero when the fluid is exposed to the atmosphere. Velocity head is
negligible if it is too big and the decrease in the level of fluid is too small, we may
consider the velocity head to be zero.

Example 12:

Pt A elev 500m

Q = 1cms D = 0.5m

Pt. C elev 200m

Pt B elev 100m

Consider a frictionless pipe.


What are the pressures at the tip of the pipe and at point C?

Q = VA
Vcπ
1= ( 0.5 )2
4
m
Vc=5.09
s

U 2A 1 P A V 2c Pc
+ + z A= + + zc
2g γ 2g γ

Page 130 of 149


5.092 Pc
500= + + 200
19.62 γ

Pc
=500−200−1.32=298.68 m
γ

N
(
Pc =298.68 m 9810
m3)=2930050.8 Pa=2930.05 KPa

Pressure at point B?

V 2A P A V 2B P B
+ + z A = + + zB
2g γ 2g γ

5.092 P B
500= + +100
19.62 γ

PB
=500−100−1.32=398.68m
γ

PB =398.68 ( 9810 )=3911050.8 Pa=3911.05 KPa

Example 13:
Pt A elev 500m

d1 = 0.5m f1 = 0.02 L1 = 300m

Pt. C elev 200m

d2 = 0.6m f = 0.018 L1 = 400m

Pt B elev 100m

Solve for the


a. Total head loss

Page 131 of 149


V 2A P A V 2B P B
+ + z A = + + z B + HLT
2g γ 2g γ
500=100+ HLT
HLT =500−100=400 m

b. Discharge

0.0826 fL Q 2 0.0826 ( 0.02 ) ( 300 ) Q 2


hf 1= 5
= 5
=15.8592 Q 2
D 0.5

0.0826 fLQ 2 0.0826 ( 0.018 ) ( 400 ) Q2


hf 2= 5
= 5
=7.6481Q2
D 0.6

HLT =15 8592Q 2+7.6481 Q 2=23.5073 Q 2=400


Q=4.12 m3 / s∨4.12 cms

c. head loss at the first pipe

hf 1=15.8592 Q2=15.8592 ( 4.12 )2=270.024 m

d. head loss at the second pipe


2 2
hf 2=7.6481 Q =7.6481 ( 4.12 ) =130.1327 m

e. velocity at the first pipe

4 Q 4 (4.12)
V 1= = =20.98 m/s
π d 2 π ( 0.5 )2

f. velocity at the second pipe

4 Q 4 (4.12) m
V 2= 2
= 2
=14.57
πd π ( 0.6 ) s

g. Pressure pt C
V 2A P A V 2C PC
+ + z A = + + z C + HLT
2g γ 2g γ

20.982 P C
500= + +200+ 270.0244
19.62 9810

PC =( 500−270.02−200−22.43 ) 9810=7.55 ( 9810 ) =74065.5 Pa=74.07 KPa

h. pressure at pt b

Page 132 of 149


V 2A P A V 2B P B
+ + z A = + + z B + HLT
2g γ 2g γ

14.572 PB
500= + +100+ 400
19.62 9810

PB =−10.85 ( 9810 )=−106434.05 Pa=−106.43 KPa

Example 14: A hospital patient needs a blood transfusion, which will be administered
through a vein in the arm via gravity IV. The physician wishes to have 500cc of the
whole blood delivered over a period of 10min by using an 18-gauge needle with a length
of 50mm and an inner diameter of 1.0mm. At what height above the arm should the bag
of blood be hung? (assume a venous blood pressure of 15mm Hg).

Given:
∆ V =500 cc=0.5 L
60 s
∆ t=10 min x =600 s
1 min
L=50 mm=0.05 m
d 1 mm
r= = =0.5 mm=0.0005 m
2 2
Pblood=15 mmHg=2.00 x 103 Pa
η=1.7 x 10−3 P

Required:
h=?

Solution:
∆ V 500 c m3 1 m3
Q=
∆t
= (
600 s 100 c m3 )
=8.33 x 10−7 cms

π r4 ∆ P
−3 −7
8 ηLQ 8 ( 1.7 x 10 ) ( 0.05 ) (8.33 x 10 )
Q= ; ∆ P= = =2884.86 Pa
8 ηL π r4 π (0.0005)4

∆ P=Pi−Po
Pi=∆ P+ Po=2884.86+ 2000=4884.86

P= ρgh

P 4884.86 N /m 3
h= = =0.47 m
ρg 1.05 x 103 ( 9.81)

Page 133 of 149


Name:
Section:

Q = 1.5cms

A B C
D = 0.25m D = 0.50m D = 0.75m

What are the velocities at points A, B, and C?

Given:
Q=2cms

Required:
V =?

Q=VA
Vπ d 2
Q=
4

4Q 4 (2) m
V A= 2
= 2
=40.74
π d π ( 0.25 ) s

4Q 4 (2) m
V B= 2
= 2
=10.19
π d π ( 0.50 ) s

Page 134 of 149


4Q 4 (2) m
V C= 2
= 2
=4.53
π d π ( 0.75 ) s

Chapter 10

Vibrations and Waves


Important Terms

Hooke’s Law - One such force is the (ideal) spring force.

Simple Harmonic Motion (SMH) – motion under the influence of the type of force
described by Hooke’s law.

Displacement – the directed distance of the object in SMH from it’s equilibrium
position.

Amplitude – the magnitude of the maximum displacement, or the maximum distance of


an object from it’s equilibrium position.

Period – the time needed to complete one cycle of motion.

Frequency – the number of cycles per second.

Hertz – the standard unit of frequency.

Equation Of Motion – an object as the equation that gives the object’s position as a
function of time.

Phase Constant – the phase constant essentially shifts the curve to match the appropriate
sinusoidal function to the motion and is determined by the initial conditions.

Damped Harmonic Motion – the amplitude and the energy of an oscillator decrease
with time.

Page 135 of 149


Wave – a regular, rhythmic disturbance in both time and space.

Wave Motion – the transfer of energy is said to take place.

Wavelength – the distance between two successive crests (or troughs).

Wave Speed – when a wave moves.

Longitudinal Wave – the particle oscillation is parallel to the direction of the wave
velocity.

Principle Of Superposition – at any time, the combined waveform of two or more


interfering waves is given by the sum of the displacement of the individual waves at each
point in the medium.
Interference – the physical addition of waves.

Constructive Interference – the vertical displacement of the two pulses are in the same
direction, and the amplitude of the combined waveform is greater than that of either
pulse.

Destructive Interference – if one pulse has a negative displacement, the two pulses tend
to cancel each other when they overlap, and the amplitude of the combined wave form is
smaller than that of either pulse.

Total Constructive Interference – at the instant these inferring waves are exactly in
phase (crest coinciding with crest), the amplitude of the combined waveform is twice that
of either individual wave.

Total Destructive Interference – when these interfering pulses are completely out of
phase (180° difference, or crest coinciding with trough), the waveforms momentarily
disappear; that is, the amplitude of the combined wave is zero.

Reflection – it occurs when a wave strikes an object or comes to a boundary of another


medium and is at least partly diverted back into the original medium.

Refraction – entering the medium obliquely (at an angle), the transmitted wave forms in
a direction different from that of the incident wave.

Dispersion – when the wave speed does depend on wavelength ( or frequency).

Diffraction – refers to the bending of waves around an edge of an object but is unrelated
to fraction.

Standing Wave – the rope shaken at the right frequency, a steady waveform, or series of
uniform loops, appears to stand in place along the rope.

Page 136 of 149


Node – some points on the rope remain stationary at all times.

Antinode – the points of maximum amplitude, where constructive interference is


greatest.

Natural (Resonant) Frequencies – the frequencies at which large-amplitude standing


waves.

Fundamental Frequency – the lowest natural frequency.

Harmonic Series – the set of frequencies.

Resonance – the condition of driving a system at a natural frequency.

Important Equations

Simple Harmonic Motion:

Hooke’s law:
F = - kx

Frequency and Period for SHM:


F = 1/ T

Total Energy of a Spring and Mass in SHM:


E = ½ kA²

Velocity of Oscillating Mass in Spring:


v = ± √ k/m (A² - x² )

Equation of Motion:

Period of Mass Oscillating on a Spring:


T = 2 √ m /k

Frequency of Object Oscillating on a Spring:


f = 1/2 √ k /m

Angular Frequency of Object Oscillating on a Spring:


ω = √ k/m

Period of a Simple Pendulum:


T = 2 √ L /g

Initial Condition and Phase:

Page 137 of 149


Displacement of a Mass in SMH:
y = A sin (ωt + ζ )
( with ζ = 0) y = A sin ωt = A sin 2ft = A sin 2t / T

Velocity of a Mass in SMH (ζ = 0):


v = ωA cos ωt

Acceleration of a Mass in SMH (ζ = 0):


A = - ω²A sin ωt = - ω²y

Wave Motion:

Wave Speed:
v = λ / T = λf
Standing Waves and Resonance:

Natural Frequencies in a Stretched String:


f = n ( v/2L) = n/2L √F / µ ( for n = 1,2,3,….)

Example 1: A block with a mass of 0.25 kg sitting on a frictionless surface is connected


to a light spring that has a spring constant of 180 N/m. If the block is displaced 15cm
from its equilibrium position and released, what are (a) the total energy of the system and
(b) the speed of the block when it is 10 cm from its equilibrium position?

Given: X 1 =10 cm=0.1 m


m 1=0.25 kg
K=180 N /m Required:
A=15 cm=0.15 m KE = ?
V =?

Solution:
KE=0.5 K A2=0.5 ( 180m N )( 0.15 m )=2.02 Nm=2.02 J
2

K ¿ 180
V=
√ m √
( A ¿ 2−x 2 ¿ )=
0.25
(0.15 2−0.12 ¿)=3 m/s ¿ ¿ ¿

Example 2: When a 0.50 kilogram mass is suspended from a spring, the spring stretches a
distance of 10 cm to a new equilibrium position. (a) What is the spring constant of the
spring? (b)The mass is then pulled down another 5.0 cm and released. What is the highest
position of the oscillating mass?

Given:

Page 138 of 149


m=0.5 kg
x=10 cm=0.1m

Required:
k =?
x p=?

Solution:
mg 0.5(9.81)
a . K= = =49.05 N /m
x 0.1

b . x p=5 cm

Example 3: A helpful older brother takes his sister to play on the swings in the park. He
pushes her from behind on each return. Assuming that the swing behaves as a simple
pendulum with a length of 2.50 m, (a) what would be the frequency of the oscillations,
and (b) what would be the interval between the brother’s pushes?

Given: Required:
L=2.5 m a . f =?
b . t =?

Solution:
1 g 1 9.81 m/s 2
a.f =

=
2π L 2π √ 2.5 m
=0.32 h z

1 1
b.T= = =3.71 s
f 0.32

Example 4: To stir a mixture of liquids consistently and gently, a biologist designs a


simple mechanical system consisting of a horizontal spring attached to a beaker on the
slippery Teflon® surface. She pours two liquids with a mass of 100 g into the beaker,
which has a mass of 50.0 g. She then pulls on the beaker, stretching the spring. On
releasing the beaker from rest (t=0), she sets the system in motion with an amplitude of
5.00 cm. To avoid spilling and to promote gentle mixing, the frequency is designed to be
exactly 0.300 Hz. (a) What is the required spring constant? (b) What is the displacement
of the beaker from equilibrium at time t = 2.00 s?

Given: f =0.3 h z
m l=100 g=0.1 kg t=2 s
mb=50 g=0.05 kg
m T =0.15 kg Required:
A=5 cm=0.05 m a . k =?

Page 139 of 149


b . x=?

Solution:
a . k =4 π 2 m f 2=4 π 2 ( 0.15 ) ( 0. 32 )=0.53 N /m

ω=2 πf =2 π ( 0.3 ) =1.88 rad /s

rad 360 °
x= ACosωt =0.05 cos 1.88 ( x
s 2 πrad )
x 2 s =−0. 04 m

Example 5: A person on a pier observes a set of incoming waves that have a sinusoidal
form with a distance of 1.6 m between the crest. If a wave laps again the pier every 4.0 s,
what are (a) the frequency and (b) the speed of the waves?

Given:
λ=1.6 m
t=4 s

Required:
a . f =?
b . v =?

Solution:
1 1
a . f = = =0.25 h z
t 4

λ 1.6
b.v= = =0.4 m/s
T 4

Example 6: A piano string with a length of 1.15 m and a mass of 20.0 g is under a tension
of 6.30 x 10³ N. (a) What is the fundamental frequency of the string when it is struck? (b)
What are the frequencies of the next harmonics?

Given:
L=1.15 m
m=20 g=0.02 kg
T =6.3 x 103 N

Required:
f 1=?
f 2=¿?

Page 140 of 149


f 3=?

Solution:
m 0.02 kg
μ= = =0.017 kg /m
L 1.15m
1 T 1 6300
f=
√ =
2 L μ 2(1.15) 0.017 √ =264.68 h z

f 2=2 f 1=2 ( 264.68 ) =529.36 h z


f 3=3 f 1=3 ( 264.68 )=794.04 h z

Example 7: You wish to raise the fundamental frequency of a guitar string from the A
note (220 Hz) below middle C to the A note (440 Hz) above middle C. Would you (a)
loosen the string to have its tension, (b) tighten the string to double the tension, (c) used
another string of the same material with half the diameter at the same tension, (d) use
another string of the material with twice the diameter at the same tension?

Given:
f 1=220 hz
f 2=440 hz

Required:
Action = ?

Solution:
f 2=f 1
440=2 ( 220 )
f 2=2 f 1

1
f∝
d
f d
2= 2 = 1
f 1 d2
d 1=2 d 2
d1
d 2=
2

Page 141 of 149


Chapter 11

Sound
Important Terms

Sound Waves - In gases and liquids (fluids) are primarily longitudinal waves. However,
sound disturbances moving through solids can have both longitudinal and transverse
components. The intermolecular interactions in solids are much stronger than in fluids
allow transverse components to propagate.

Audible Region of the Sound Frequency Spectrum - Are characteristics of the ear limit
the perception of sound. Only sound waves with frequencies between about 20 Hz and 20
kHz (kilohertz) initiate nerve impulse that are interpreted by the human brain as sound.

Infrasonic Region are Frequencies lower than 20 Hz. Waves in this region, which we
can’t hear, are found in nature. Longitudinal waves generated by earthquakes have
infrasonic frequencies, and we use these waves to study the Earth’s interior. Infrasonic
waves, or infrasound, are also generated by wind and weather patterns.

Ultrasonic Region is above 20 kHz. Ultrasonic waves can b generated by high-


frequency vibrations in crystals. Ultrasonic waves, or Ultrasound, cannot be detected by
humans but can be by other Animals.

Intensity is a rate of the energy transfer, which is the energy transported per unit time
across a unit area.

Loudness is sound intensity perceived by the ear. On the average, the human ear can
-12
detect sound waves (at 1 kHz) with an intensity as low as 10 W/m².

Bel (B) is the exponent of the power of ten in the final log term is taken to have a unit.

Page 142 of 149


Decibel (Db) is a tenth of a bel. The 0 to 12 B range corresponds to 0 to 120 dB.

Constructive Inference is the waves meet in a region where they are exactly in phase.

Destructive Interference is the waves meet such that the crest of one coincides out.

Phase Difference ( ) is related to the Path-Length Difference ( ) by the


simple relationship.
Beats - The ear senses pulsations in loudness

The maximum amplitude is 2A (at the point where the maxima of the two original waves
interfere constructively). Detailed mathematics shows that a listener will hear these beats
at a frequency called the Beats Frequency (fb).

If you stand along the highway and a car or truck approaches you with its horn blowing,
the Pitch (the perceived frequency) of the sound is higher as the vehicle approaches and
lower as it recedes.

A variation in the perceived sound frequency due to the motion of the sound source is an
example of the Doppler Effect

Sonic Boom is when pressure ridge passes over an observer on the ground, large
concentration of energy produces.

Mach Number (M) is inverse ratio of the speed, named after Ernst Mach (1838 – 1916),
an Austrian Physicist, who use it in studying supersonics.

The Quality of a tone is the characteristic that enables it to be distinguished from another
of basically the same intensity and frequency.

Important Equations

Speed of Sound (in meters per second):


v = (331 + 0.6Tc) m/s

Intensity of a Point Source:


1
I² / I = P/4pR2
I² / I1 = (R1 / R2)²

Intensity Level (in decibels)


I
β=10 log ( )
Io

Page 143 of 149


where I = 10-12 W/m²
º

Phase Difference (where L is path-length difference):


Dθ = 2p / λ (DL)

Condition for destructive interference:


DL = nλ (n = 0,1,2,3,……)

Condition for Destructive Inference:


DL = m(λ / 2) (m = 1,3,5,……)

Beat frequency
fb | f1 – f2|

Doppler Effect
v
( )
f o= f
v ± vs s

- for moving toward stationary observer


+ for source away from stationary observer

Where: vs = speed of source


v = Speed of sound

f o= ( v ±vv ) f
o
s

+ observer moving toward stationary source


- for observer away from stationary source

where v = speed of observer


°
v = speed of sound

Mach number
vs
M=
v

Natural Frequency of Organ Pipe Open on Both Ends:


fn = n (v / 2L) = nf1
(n = 1,2,3,………)

Natural Frequency of Organ Pipe Closed on One End:

Page 144 of 149


f m=m ( 4vL )=m f 1

(m = 1,2,3,……)

Example 1: From their material properties, find the values of speeds of sound in (a) a
solid copper rod, (b) liquid water, and (c) air at room temperature (20°).

Given: T =20 ° C
∝Cu=11 . 0 x 10 10 N /m 2
ρCu =8 . 9 x 103 kg /m3 Required:
a . V Cu =?
β H O =2. 2 x 109 N /m 2
2
b . V H O =?
ρ H O =1000 kg /m3
2
2

c .V a=?

Solution:
∝ 11 x 1010 N m2
a . V Cu =
ρ
=
√ √ 3
8 . 9 x 10 kg /m 3
=3 . 52 x 103 m/ s

β 2 . 2 x 10 9 N /m2
b . V H O= =
2
ρ √ √ 1000 kg /m 3
=1.45 x 103 m/s

c .V a=331+0.6 T =331+0.6 ( 20 )=343m/ s

Example 2: On a cool October afternoon (air temperature = 15°C), from your seat in the
centerfield stands 113 m from first base, you witness the play that will decide the World
Series. You see runner’s foot touch the Bag; half a second later, straining your ears, you
hear the faint thud of the ball in the first baseman’s glove. The umpire signals safe; half
the fans boo loudly. As a student of physics, you make the call – did the ump blow it?

Given:
T c =15 °C

Page 145 of 149


d=113 m

Required:
Correct call?

Solution:
V =331+ 0.6T =331+ 0.6 (15 )=340 m/s

d d 113
V = ; t= = =0.033 s
t V 340

d d 113
C= ; t= = =3.77 x 10−7 s
t C 3.0 x 108

-12
Example 3: What are the intensity levels of sounds with intensities of (a) 10 W/m²
and (b) 5.0 x 10-6 W/m²?

Given:
a . I =1 x 10−12 W /m2
b . I =5 x 10−6 W /m2

Required:
β=?

Solution:
I 1 x 10−12
a . β=10 log
( )
Io
=10 log( )
1 x 10−12
=0 dB

I 5 x 10−6
b . β=10 log
( )
Io
=10 log( )
1 x 10−12
=66.99 dB

Example 4: What is the difference in the intensity levels if the intensity of a Sound is
doubled? (b) By what factors does the intensity increase for intensity level differences of
10 dB and 20 dB?

Given:
I 1=I 2 Required:
∆ B=10 dB a . ∆ B=?
∆ B=20 dB b . I 1 =?
Solution:

Page 146 of 149


I2 2 I1
a . ∆ B=10 log ( )
I1
=10 log ( )
I1
=10 log 2=3.01 dB

I I
β=10 log ( )
Io
β=10 log ( )
Io

10=10 log
( 1 x 10I ) −12
20=10 log
( 1 x 10I ) −12

1=log
( 1 x 10I )−12
2=log
( 1 x 10I )−12

I I
101= 102=
1 x 10−12 1 x 10−12
I =101 ( 1 x 10−12 )=1 x 10−11 W /m 2 I =102 ( 1 x 10−12 )=1 x 10−10 W /m 2

Example 5: Sitting at a sidewalk restaurant table, a friend talks to you in normal


conversation (60 dB) At the same time, the intensity level of the street traffic reaching
you is also 60 dB. What is the total intensity level of combined sound?

Given:
β 1=60 dB Required:
β 2=60 dB β 2=?

Solution:
β=10 log ( II )
o

60=10 log
( 1 x 10I ) −12

I
6=log
( 1 x 10 ) −12

I
106 =
1 x 10−12
I =106 ( 1 x 10−12 )=1 x 10−6 W /m 2

β T =I 1 + I 2=1 x 10−6 +1 x 10−6=2 x 10−6 W /m2

I
β=10 log ( )
Io

Page 147 of 149


2 x 10−6
β=10 log ( 1 x 10−12)=63.01 dB

Example 6: At an open-air concert on a hot Day (air temperature of 25°C), you sit 7.00
and 9.10m, respectively, from a pair of speakers, one at each side of the stage. A
musician, warming up, plays a single 494-hertz tone. What do you hear? (Consider the
speakers to be point sources.)

Given: T =25 ℃
d 1=7 m
d 2=9.10 m Required:
∆ L=?

Solution:
V =331+ 0.6T =331+ 0.6 ( 25 )=346 m/s
v 346 m/ s
d= = =0.7 m=700 mm
f 494 s
λ
d 1=7000 ( )
700
=10 λ

λ
d 2=9100 ( )
700
=13 λ

∆ L=d 2−d 1=13 λ−10 λ=3 λ


Example 7: As a truck traveling at 96 km/h approaches and passes a person standing
along the highway, the driver sounds the horn. If the horn has a frequency of 400 Hz,
what are the frequencies of the sound waves heard by the person (a) as the truck
approaches and (b) after it has passed? (Assume that the speed of sound is 346 m/s.)

Given:
V o =0 m/s
km 1000 m 1 hr
V s =96 x x =26.67 m/ s
hr 1 km 3600 s
V =346 m/s
f s=400 hz

Required:
f o=?

Solution:
v 346
( ) (
a . f o=
v±vs
f s=
346−26.67 )
( 400 )=433.31 hz

( v ±vv ) f =( 346+26.67
b . f o=
s
s
346
) ( 400)=371.37 hz
Page 148 of 149
Page 149 of 149

You might also like